Exclusive HOLI Discounts!
Get Courses and Combos at Upto 50% OFF!
Upgrad
LCI Learning

Share on Facebook

Share on Twitter

Share on LinkedIn

Share on Email

Share More

Victim of Greed (Worker)     29 July 2013

Are household items also called as stridhan ?

Hello Experts,

 

Would like to ask whether Household Items which I had bought from my own money in the house when we were living together, are also called as Stridhan ? In her e-petition for return of Stridhan, She had mentioned list of articles and Jewellery lying with me. I am aware that Jewellery is called as Stridhan but what about household items which were not gifted to her as they were been used only in the house.

Please advice me.



Learning

 12 Replies

Suri.Sravan Kumar (senior)     29 July 2013

pl refer to the reply given by A.V.Vishal Advocate on this type of query. This will cover your query also.( for his reply pl type stridhan on google search)

Adv k . mahesh (advocate)     29 July 2013

if you have any proof of bills or any sought of evidences to show that it was purchased after marriage and not given at the time of marriage then it will not come under streedhan 

go to this thread for streedhan

https://www.lawyersclubindia.com/forum/How-a-bride-can-secure-her-streedhan-after-marriage--24728.asp#.UfZZVdKbfz4

1 Like

Big Iron (Officer)     30 July 2013

I am not an advocate but isn't other side resposible to show the evidence first that they had given those household items?

I mean I can have n number of things at my home and they cant say they had given this and that without any proof?

 

"Victim of marriage", I've not read your previous posts but let me tell you one thing, this all is crap. Indian courts spl in matrimonial issues are just deaf n dumb.

Dont worry about anythng but keeping yourself and your family happy.

Victim of Greed (Worker)     31 July 2013

Thank you Mr. Mahesh,

 

I just read the link you have sent but it has not mentioned that any gifts or any immovable/movable property gifted after marriage is not called stridhan.

Household appliances such as Washing machine and Air condition were bought by me after marriage and had paid by cash but the bill was in her name. She is claiming for everything but she does not have the bills of anyone.

Please help me !!!!!


(Guest)

Gifts etc [jwellery, ac, bus, car, window ac, split ac, almirah, washing machine, dishwasher etc etc] given at the time of marriage = stridhan. After marriage anything husband has brought, then it is not stridhan. You need not give back such items, if your wife has bills also she cannot claim it as stridhan. simple.


Nadeem Qureshi (Advocate/ nadeemqureshi1@gmail.com)     31 July 2013

1 Like

Nadeem Qureshi (Advocate/ nadeemqureshi1@gmail.com)     31 July 2013

Supreme Court of India
Pratibha Rani vs Suraj Kumar & Anr on 12 March, 1985
Equivalent citations: 1985 AIR 628, 1985 SCR (3) 191
Bench: Fazalali, S Murtaza

PETITIONER:

PRATIBHA RANI

Vs.

RESPONDENT:

SURAJ KUMAR & ANR.

DATE OF JUDGMENT12/03/1985

BENCH:

FAZALALI, SYED MURTAZA

BENCH:

FAZALALI, SYED MURTAZA

MUKHARJI, SABYASACHI (J)

VARADARAJAN, A. (J)

CITATION:

1985 AIR 628 1985 SCR (3) 191

1985 SCC (2) 370 1985 SCALE (1)458

CITATOR INFO :

RF 1986 SC 833 (50)

RF 1992 SC 604 (103)

ACT:

Nature, character and concomitants of stridhan-Right of exclusive ownership over the stridhan during coverture- Whether the dowry/stridhan given to a wife and her exclusive property becomes a joint property/partnership property by a fiction of being placed in the custody of her husband and her relations, the moment a married woman enters her matrimonial home-Indian Partnership Act, 1932, section 4. Dowry Prohibition Act (28 of 1961) sections 2 Hindu Marriage Act, 1955 section 27 Hindu Succession Act Section 14 Indian Penal Code, sections 405, 406 and 482.

Remedies open under law-Whether criminal remedy is barred when civil remedy is available simultaneously. Entrustment-Charge Or Criminal breach of trust by wife against her husband and his close relations maintainability- Essential ingredients of an offence section 405/406 Indian Penal Code.

Inherent powers of the High Court to quash a First Information Report on a complaint under section 482' the Code of Criminal Procedure, 1973 (Act 11 of 1974), explained.

HEADNOTE:

The appellant Pratibha Rani, the estranged wife of the first respondent Suraj Kumar, filed a criminal complaint against her husband, his father, his three brothers and a brother-in-law in the court of the Additional Chief Judicial Magistrate, Ludhiana, alleging; (i) that she was married to the first respondent at Ludhiana on 4 2. 1972 according to Hindu rites and customs; (ii) that the aforesaid persons, namely, father, brothers and brother-in-law of the first respondent attended the marriage and demanded dowry from the appellants' parents as consideration for the marriage; (iii) that the dowry articles mentioned in the list worth Rs 60,000 in the form of gold ornaments, clothes and other valuables were given and entrusted to the respondents and four others at Ludhiana at the time of 'doli' on 5. 2. 1972 in the presence of Kapur Chand Jain and six others; (iv) that all the six respondents, from the time of marriage started teasing, harassing and beating her and they kept her without even food to extract more money from her parents; (v) that they turned out the appellant with her children in the beginnings of 1977 (vi) that after a great deal of persuasion and intervention by Panchayatdars, respondent No. I came

192

to Ludhiana and took her to his house, after giving an undertaking in writing on 21. 6. 1977 not to misbehave with and not to maltreat the appellant and her children; (vii) that after some time all the respondents in the Complaint not only started again maltreating the appellant and misbehaving with her, but also brought the appellant at 4.30 a.m. On 11.12.80 and left her near Kailash Cinema Chowk, (viii) that the articles (the stridhana) mentioned in the list appended to the complaint were never given by the respondents to the appellant for her use but were retained by them illegally and with the dishonest intention of causing wrongful gain to themselves and wrongful loss to the appellant y (ix) that when the appellants' husband and his brother, Vishwinder Kumar, respondent 1 and 5 in the complaint, came to Ludhiana on 10 2.81 to attend the proceedings started by the appellant under section 125 Criminal Penal Code her parents persuaded them to return the articles entrusted to them at the time of the marriage but they flatly refused to comply with that demand; (x) that the articles have not been returned in spite of service of notice dated 17.12.81 on the first respondent; (xi) that the respondents in the complaint have dishonestly, thus, converted the articles belonging to the appellant for their use in violation of the instructions of the appellants' parents given at the time of the marriage to give the articles for the appellants' use and that (xii) they individually and jointly committed the offences under sections 405 and 406 Indian Penal Code.

Thereupon respondent No. 1 filed Criminal Misc. Application No. 4876 of 1981 in the Punjab and Haryana High Court under section 482 of the Code of Criminal Procedure for quashing the criminal proceedings and the complaint taken on file by the Additional Chief Judicial Magistrate, Ludhiana under section 406 IPC and his order summoning them.A Learned Single Judge of the High Court relying strongly upon the observations made by a Full Bench of that High Court in Vinod Kumar Sethi & Ors. v. State of Punjab & Ors. reported in AIR 1982 Punjab 372 allowed the petition and quashed the proceedings arising out of the appellants' complaint, observing that the allegations in the appellants' complaint are similar to the one in that case and therefore, fully covered by the ratio in that decision. Hence the appeal by special leave.

Allowing the appeal, the Court,

^

HELD; (Per E.lzal Ali, J.) (on behalf of Sabyasachi Mukharji, J. and himself)

1.1 The stridhan property of a married woman cannot acquire the character of a joint property of both the spouses as soon as she enters her matrimonial home so as to eliminate the application of section 406 IPC. The position of stridhan of a Hindu married woman's property during coverture is absolutely clear and unambiguous; she is the absolute owner of such property and can deal with it in any manner she likes-She may spend the whole of it or give it away at her own pleasure by gift or will without any reference to her husband. The entrustment to the husband of the stridhan property is just like something which the wife keeps in a bank and can withdraw any amount when ever she likes without any hitch or hindrance. Ordinarily, the husband has no right or interest in it with the sole exception that in times of extreme distress, as 193

in famine, illness or the like, the husband can utilize it but he is morally bound to restore it or its value when he is able to do so. This right is purely personal to the husband and the property so received by him in marriage cannot be proceeded against even in execution of a decree for debt. [206F; 201D-E]

Suraj Kumar & Anr. v. Pratibha Rani, Criminal Misc. Petition No. 4876 of 1981 Punjab & Haryana High Court reversed.

Vinod Kumar Sethi & Ors. v. State of Punjab & Anr. AIR 1982 Punjab 372; Surinder Mohan v. Smt Kiran Saini, 1977 Chandigarh Law Reporter 212; Kailash Vati v. Ayodhya Parkash, ILR (1977) 1 Punjab & Haryana 642 (FB) overruled. 1.2 A perusal of the allegations made in the complaint undoubtedly makes out a positive case of the accused having dishonestly misappropriated the articles handed over to the n in a fiduciary capacity. To characterise such an entrustment as a joint custody or property given to the husband and the parents is wholly unintelligible.A perusal of the list reveals that so far as the jewellery and clothes, blouses, nighties and gowns are concerned they could be used only by the wife and were her stridhan. By no stretch of imagination could it be said that the [ornaments and sarees and other articles mentioned above could also be used by the husband. If, therefore, despite demands these articles were refused to be returned to the wife by the husband and his parents, it amounted to an offence of criminal breach of trust. All the ingredients of an offence under section 405 IPC were pleaded and a prima facie case for summoning the accused was made out. In such circumstances, the complaint should have been given an opportunity by the High Court to prove her case rather than quashing the complaint. Such an exercise of jurisdiction by the High Court under section 482 Cr. P. is totally unwarranted by law. [203A; 204B-D; 203B-C] 2.1 Criminal law and matrimonial home are not strangers. Crimes committed in matrimonial home are as much punishable as anywhere else. The mere factum of the husband and wife living together does not entitle either of them to commit a breach of criminal law and if one does then he/she will be liable for all the consequences of such breach. In the case of stridhan properly also, the title of which always remains with the wife though possession of the same may sometimes be with the husband or other members of his family, if the husband or any other member of his family commits such an offence, they will be liable to punishment for the offence of criminal breach of trust under sections 405 and 406 IPC. Just as a newly married woman living in the same house and under the same roof cannot be expected to keep her personal property or belongings like jewellery, clothing, etc. under her own lock and key thus showing a spirit of distrust to the husband at the very behest, a husband cannot be permitted to cast his covetous eyes on the absolute and personal property of his wife merely because it is kept in his custody, thereby reducing the custody to a legal farce. On the other hand, even if the personal property of the wife is jointly kept it would be deemed to be expressly or impleedly kept in the custody of the husband and i f he dishonestly misappropriates or refuses 194

to return the same, he is certainly guilty of criminal breach of trust, and there can be no escape from this legal consequence. [207E-G; 208B-Cl

2.2 It is an anathema to suppose that when a civil remedy is available, a criminal prosecution is completely barred. The two remedies-are under civil law and the other under criminal law are not mutually exclusive but coextensive and essentially differ in their content and consequences. Therefore, it cannot be said that, if the husband dishonestly misappropriates the stridhan property of his wife though kept in his custody, that would not par prosecution under section 406 IPC or render the ingredients of section 405 IPC nugatory or abortive. To say that because the stridhan of a married woman is kept in the custody of her husband no action against him can be taken as no offence is committed is to override and distort the real intent of law. [208E-F]

3.1 Neither section 27 of the Hindu Marriage Act nor section 14 of the Hindu Succession Act, go to the extent of providing that the claim of a woman on the basis of stridhan is completely abolished, or that a remedy under the criminal law for breach of trust is taken away. All that the two sections, provide is that if the husband refuses to return the stridhan property of his wife, it will be open to the wife to recover the same by a properly constituted suit. [204G-H; 205A]

3.2 Section 27 of the Hindu Marriage Act merely provides for- an alternate remedy and does not touch or affect in any way the criminal liability of the husband in case it is proved that he has dishonestly misappropriated the stridhan of his wife. It cannot also be spelt out from any textbook or the sastric law of the Hindus that these two Acts take away the stridhan right of a woman-at the most these Acts merely modify the concept of stridhan. [205C-D]

Bhai Sher Jang Singh & Anr. v. Smt. Virinder Kaur, 1979 Criminal Law Journal 493 approved.

Surinder Mohan v. Smt. Kiran Saini, 1977 Chandigarh Law Reporter 212 over ruled.

4.1 It is neither appropriate nor apposite to import the concept of partner ship of husband and wife for the simple reason that the concept of partnership is entirely different from that of the husbands' keeping the stridhan in his custody. From the definition of the partnership in section 4 of the Indian Partnership Act, it is manifest that in a partnership the wife must by some clear and specific act indicate that the stridhan which has been entrusted to the husband is to be used for a partnership business and the losses of the firm, if any, would have to be shared by both.A pure and simple act of entrustment of the stridhan to the husband does not attract any of the essential ingredients of a partnership as defined in the Partnership Act. When the essential conditions of a partnership do not exist. the mere factum of entrustment of stridhan would not constitute any co-ownership or legal partnership, There is also no

195

question of the wife, constituting herself a partner with her husband merely by allowing him to keep the article or money in his custody. Further, in this case, there is, neither any pleading nor any allegation that after her marriage, the appellant transferred all her properties to her husband for carrying on a partnership business in accordance with the provisions of the Partnership Act. Therefore, a criminal prosecution under section 406 IPC is maintainable.

[209E; 210B-C; G; 211C-D]

Vinod Kumar Sethi & Ors. v. State of Punjab & Anr. AIR 1982 Punjab 372; Surinder Mohan etc. V. Smt. Kiran Saini, 1977 Chandigarh Law Reporter 212; Kailash Vati v. Ayodhya Parkash, ILR (1973) 1 Punjab & Haryana, P 612; Kailash Nath Agarwal & Ors. v. Prem Pal Agarwal & Anr. Crl. Misc. case No. 676 of 1981 connected with Crl. Misc. case No. 2753 of 1981 decided on 22.12.83 Allahabad High Court overruled. In the instant case, however, there is neither any allegation nor anything in the complaint to show that when the wife entered her matrimonial home she had entrusted property to her husband so as to make him part owner of the same. Therefore, the question Or the husband having dominion over the property does not at all arise. In fact the wife has nothing to do With the partnership, if any and the husband is a pure and simple custodian of the property and cannot use the Same for any purposed without her consent. [210E-F]

The concept of stridhan property of a married woman becoming joint property of both the spouses as soon as she enters her matrimonial home and continues to be so until she remains there or even if there is a break in the matrimonial alliance, is in direct contravention of Hindu law of Sadayika which has been administered since more than a century by High Court, Privy Council and also the Supreme Court. [212C-D]

4.2 The Full Bench decision in Vinod Kumar's case would not only render the provisions of section 406 IPC inapplicable and nugatory even if the husband has the audacity or the importunity of refusing to return the stridhan of his wife, but also be in direct contravention of a long course of decisions of Supreme Court on the ingredients of section 405 IPC. [212A-B]

By a pure and simple figment of the fertile imaginations, the Judges in the Vinod Kamat's case seem to have rewritten the law of criminal breach of trust contained in sections 405 and 406 IPC so as to carve out an imaginary exception to the application of the Penal Code- more tragic consequence of the view taken by the High Court is that even if there is a break in the matrimonial alliance and the wife wants her husband to return her exclusive property and he refuses lo return even then the provisions of section 406 IPC would not apply. It is an extreme travesty of justice for a court to say that whenever a married demands her stridhan property from her husband she should be driven to the dilatory process of a civil court and her husband would be debarred from being prosecuted by a criminal court. By a strange and ingenious process of holding that such an act of a husband does not attract the provisions of the 196

Penal Code, as the property being joint there is no question of the husband being a trustee or holding the same in a fiduciary capacity. Such a view is not only contradictory but-what the High Court has said before regarding the applicability of section 27 of the Hindu Marriage Act and the nature of stridhan-is also neither in consonance with logic and reason nor with the express provisions of the Penal Code and seems to be inspired by a spirit of male chauvinism so as to exclude the husband from criminal liability merely because his wife has refused to live in her matrimonial home. The High Court, functioning in a civilised and socialistic society such as ours cannot play such a havoc with judicial interpretation of an important branch of law. The High Court cannot make a complete volte-face by holding that these very properties after marriage become joint property of both the spouses. The High Court has not realised that the theory or philosophy of matrimonial home propounded by it stands directly contradicted by its own observations.

[212D-H; 213A; H; 214A]

4.3 The fundamental core of the offence of criminal breach of trust is that a property must be entrusted and the dominion of the property should be given to the trustee. In the present case, all these conditions, even according to the findings of the High Court though not its conclusions are clearly established.

[217C]

Chelloor Manaklal Narayan Ittiravi Nambudiri v. State of Travancore; AIR 1953 SC478; Jaswantrai Manilal Akhaney v. State of Bombay, [1956] SCR 483; State of Gujarat v. Jaswantlal Nathalal[1968] 2 SCR 408; Sushil Kumar Gupta v. Joy Shankar Bhattacharjee, AIR 1971 SC 1543; Superintendent JUDGMENT:

[4] SCC 230 referred to.

Harihar Prasad Dubey v- Tulsi Das Mundhra & Ors. AIR 1949 Calcutta 207; Akharbhai Nasarali v. Md. Hussain Bhai AIR 1961 MP 37; Basudeb Patra v. Kana. Lal Haldar, AIR 1949 Calcutta 207, Bhai Sher Jang Singh and Anr. v. Smt. Virinder Kaur, 1979 Crl. L-J. 493; Avtar Singh and Anr v. Kirpal Kaur, Crl. Misc. No. 2144 of 1979 and Cr l Misc. No. 2145 of 1979 approved.

Vinod Kumar Sethi & Ors. v- State of Punjab and Anr. ATR 1982 Punjab 372; Surinder Mohan etc. v. Smt. Kiran Saini, 1977 Chandigarh Law Reporter 212; Kailash Nath Agarwal & Ors- v. Prem Pal Agarwal & Anr. Crl. Misc. Case No. 676 of 1981 connected with Crl. Misc. case No. 2753 of 1981, Allahabad High Court: Kailash Vati v. Ayodhya Parkash, ILR (1977) 1 Punjab d: Haryana 642 overruled.

5. For the purpose of exercising its power under section 482 Cr. PC to quash a First Information Report or a complaint the High Court would have to proceed entirely on the basis of the allegations made in the complaint or the documents accompanying the same per se. It has no jurisdiction to examine the correctness or otherwise of the allegations. In case no offence is committed on the allegation and the ingredients of section 405 and 406 IPC are not made out, the High Court would be justified in quashing the proceedings. In the -present case, the allegations are both clear, specific and unambiguous and 197

therefore, the complaint should have been given a chance to prove her case. It is, of course open to the accused at the trial to take whatever defences that were open to him or her but that stage had not yet come and therefore, the High Court was totally ill-advised to speculate on the merits of the case at that stage and quash the proceedings. Since all the facts stated in the complaint constituted an offence under section 406 IPC, the appellant cannot be denied the right to prove her case at the trial by pre-empting it the very behest by the order passed by the High Court. [223D-H; 224D-E-]

Vinod Kumar Sethi & Ors. v. State of Punjab & Anr, AIR 1982 Punjab 372, over-led.

L.V. Jadhav v. Shakarrao Abasaheb Pawar & Ors. AIR 1983 SC 1219; Smt. Nagawa v. Veeranna Shivalingappa Konjalgi & ors. [1976] Supp. SCR 123 applied.

OBSERVATION

(It is surprising to find that so deeply drowned and inherently engrossed are some of the High Courts in the concept of matrimonial home qua the stridhan property- of a married woman that they simply refuse to believe that such properties are meant for the exclusive use of the wife and could also be legally entrusted to the husband or his relations. Thus, if the husband or his relations misappropriate the same and refuse to hand it over to the wife and convert them to their own use and even though these facts arc clearly alleged in a complaint for an offence under section 405/406 IPC, some courts take the view that the complaint is not maintainable. Thus even when clear and specific allegations are made in the complaint that such properties were entrusted to the husband, they refuse to believe these hard facts and brush them aside on the ground that they are vague, and completely shut their eyes to the fact that the husband could also be guilty under section 405/406 IPC in view of the clear allegations made in the complaint. In other words, the High Courts simply refuse to believe that there can be any such entrustment and even if it is so, no offence is committed. Such an approach amounts to a serious distortion of the criminal law, resulting in perpetrating grave and substantial miscarriage of justice to the wife at the hands of the High Courts. The Supreme Court cannot continuance such a wrong and perverse approach.) [224G-H, 225A-C]

Per A. Varadarajan, J. (dissenting)

1.1 In the absence of a separate agreement and specific entrustment by the wife to the husband and or his relations and vice versa of the property of the husband to the wife and or her relation, it would not be possible to draw an inference of entrustment of custody or dominion over the property of one spouse to the other and his or her relations so as to attract the stringent provisions of section 406 IPC. The offence of criminal break of trust is cognizable and non-bailable and punishable with imprisonment for a term of three years or with fine or with both. In the absence of such a separate agreement for specific entrustment of the property of either spouse the appropriate remedy would appear to be by way of a civil suit where there is scope for the parties to the marriage coming together at the instance of relations, elders and well- wishers and patching up their differences. [241G-H; 242A] 198

1.2 Entertaining complaints of the irate wife or husband against the husband or wife without even an allegation of a specific and separate agreement constituting entrustment of the property of the wife of the husband would have disastrous effects and consequences on the peace and harmony which ought to prevail in matrimonial homes. [242B] 1.3 The fact that no instance of any case of successful prosecution of the husband or wife at the instance of the wife or the husband could be brought to the notice of the Supreme Court in the course of the arguments in this appeal would show that the spouses had not lightly rushed in the past to criminal courts with complaints of criminal breach of trust against the other spouses though in the day-to-day life. There must have been numerous instance where the wife had used the property or cash of the husband for purposes different from the one for which they were given by the husband to be applied by the wife and vice-versa. Therefore, the minimum requirement in such cases is a specific separate agreement whereby the property of the wife or husband was entrusted to the husband or wife and or his or her close relations. In the absence of such a specific separate agreement in the present case the complaint was rightly quashed. [242D-F]

&

CRIMINAL APPELLATE JURISDICTION: Criminal Appeal No. 684 of 1982

From the judgment and order dt. the 31st May, 1982 of the High Court of Punjab & Haryana at Chandigarh in Crl. Misc. No. 4876M/81.

V.C. Mahajan, and N.S. Das Bahl for the Appellant. Altat Ahamed for the Respondents.

Mrs. U. Kapoor for the Intervener.

T The following Judgments were delivered FAZAL ALI, J. Sometimes the law which is meant to impart justice and fair play to the citizens or people of the count is so torn and twisted by a morbid interpretative process that instead of giving haven to the disappointed and dejected litigants it negatives their well established rights in law. The present case reveals the sad story of a helpless married woman who, having been turned out by her husband without returning her ornaments, money and clothes despite repeated demands, and dishonestly misappropriating the same, seems to have got some relief by the court of the first instance but to her utter dismay and disappointment when she moved the High Court she was forced like a dumb- driven cattle to seek the dilatory remedy of a civil suit- such was the strange and harsh approach of the High Court, with due respect, which seems to have shed all the norms of justice and fair play. Even so, the High Court is not much to be blamed because in the process of following precedents or decisions of doubtful validity of some courts, it tried to follow suit. It may be stated that even the old 199

classic Hindu law jurists and celebrated sages conceded certain substantial rights to the women, one of which was - what is called Saudayika or stridhan, with which we are concerned here.

This now brings us to a brief discussion of the nature, character and concomitants of stridhan. In the instant case, we are mainly concerned with that part of stridhan which is the absolute property of a married women during coverture. Sir Gooroodas Banerjee in 'Hindu Law of Marriage and Stridhana' while describing the nature of stridhan quoted Katyayana thus:

"Neither the husband, nor the son, nor the father, nor the brother, has power to use or to alien the legal property of a woman. And if any of them shall consume such property against her own consent he shall be compelled to pay its value with interest to her, and shall also pay a fine to the king... Whatever she has put amicably into the hands of her husband afflicted by disease, suffering from disease, or sorely pressed by creditors, he should repay that by his own freewill. " (P.341)

At another place while referring to the nature of a husband's rights over stridhan during coverture, the author referring to Manu says thus:

" .. and by the law as expounded by the commentators of the different schools, the unqualified dominion of the husband is limited to only some descripttions of the wife's property, while as regards the rest he is allowed only a qualified right of use under certain circumstances specifically defined." (p.340)

Similarly, while describing the nature of stridhan generally, which is known as saudayika, the author says thus:

"First, take the case of property obtained by gift. Gifts of affectionate kinderd, which are known by the name saudayika stridhana, constitute a woman's absolute property, which she has at all times independent power to alienate, and over which her husband has only a qualified right, -namely, the right of use in times of distress."

200

The entire classical text on the subject has been summarised by N.R. Raghavachariar in 'Hindu Law' (5th Edn) at page 533 (section 487) where the following statement is made:

"487. Powers During Coverture.

Saudayika, meaning the gift of affectionate kindred, includes both Yautaka or gifts received at the time of marriage as well as its negative Ayautaka. In respect of such property, whether given by gift or will, she is the absolute owner and can deal with it in any way she likes. She may spend, sell or give it away at her own pleasure by gift or will without reference to her husband and property acquired by it is equally subject to such rights. Ordinarily, the husband has no manner of right or interest in it. But in times of extreme distress, as in famine, illness or imprisonment, or for the performance of indispensable duty the husband can take and utilise it for his personal purposes, though even then he is morally bound to restore it or its value when able to do so. But this right is purely personal to him and cannot be availed of by a holder of a decree against the husband, and if the husband dies with out utilising the property for the liquidation of his debts, his creditors cannot claim to proceed against it in the place of her husband."

To the same effect is Maines' treatise on Hindu Law at page 728. The characteristics of Saudayika have also been spelt out by Mulla's Hindu law at page 168 (section 113) which gives a complete list of the stridhan property of a woman both before and during coverture, which may be extracted thus:

"113. Manu enumerates six kinds of stridhana:

1. Gifts made before the nuptial fire, explained by Katyayana to mean gifts made at the time of marriage before the fire which is the witness of the nuptial (adhyagni).

2. Gifts made at the bridal procession, that is, says Katyayana, while the bride is being led from the residence of her parents to that of her husband 201

(adhyavanhanika)

3. Gifts made in token of love, that is, says Katyayana, those made through affection by her father-in-law and mother-in-law (pritidatta), and those made at time the of her making obeisance at the feet of elders (padavan danika).

4. Gifts made by father.

5. Gifts made by mother.

6. Gifts made by a brother."

It is, therefore, manifest that the position of stridhan of a Hindu married woman's property during coverture is absolutely clear and unambiguous; she is the absolute owner of such property and can deal with it in any manner she likes - she may spend the whole of it or give it away at her own pleasure by gift or will without any reference to her husband. Ordinarily, the husband has no right or interest in it with the sole exception that in times of extreme distress, as in famine illness or the like, the husband can utilise it but he is morally bound to restore it or its value when he is able to do so. It may be further noted that this right is purely personal to the husband and the property so received by him in marriage cannot be proceeded against even in execution of a decree for debt.

Such being the nature and character of stridhan of a woman, it is difficult to countenance the view of the Punjab & Haryana High Court in Vinod Kumar Sethi & Ors. v. State of Punjab & An.(l) that the stridhan property of a married woman becomes a joint property as soon as she enters her matrimonial home. We shall deal with this aspect of the matter a little later.

We would first like to narrate the facts of the case to show how the complaint filed by the appellant was wrongly quashed by the High Court. The general allegations made in the complaint may be summarised as follows:- (1) AIR 1982 Punjab 372-

202

The complainant was married to Suraj Kumar, Accused No. 1 (respondent) on 4.2.72 at Ludhiana according to Hindu rites and customs in the presence of respectable persons. Accused No.2 was the father and accused Nos.3 to 5 were brothers and No.6 was brother-in-law of accused No.; It is further alleged that all the accused attended and actively participated in the marriage of the complainant and demanded dowry. The must important allegation made by the appellant was that her parents and relatives gave by way of dowry articles worth Rs. 60,000/- inclusive of gold ornaments, clothes and other things which were entrusted to accused Nos.1 to 6 on 5.2.72 which were taken into possession by them. Soon after the marriage, accused No. 1 started harassing, teasing and beating the complainant and ultimately turned her out alongwith her children sometime in the year 1977. It was avered in para 4 of the complaint that accused never returned the articles to her, the relevant portion of the allegations may be extracted thus:- "The articles above-mentioned were never given by the accused to the complainant for her use and possession of the same was illegally, dishonestly and malafidely retained by the accused in order to make a wrongful gain to them selves and wrongful loss to the complainant.

The accused refused to give the entrusted articles of dowry, which were the stridhan of the complainant. On 10.2.1981 when the accused Nos. 1 to 5 came to Ludhiana to attend the proceeding u/s 125 Cr.P.C., filed by the complainant in the Court of Shri S.S. Tiwana, they were persuaded by the parents of the complainant to send the articles entrusted to them at the time of marriage but they gave flat refusal to its notice which was served upon the accused No.1 which was dated 17.12.80, but to no effect. The accused have thus dishonestly used and converted the articles aforementioned to their own use, who are still in possession of the same in violation of the direction given by the parents of complainant. The parents of the complainant directed the accused at the time of marriage to give the articles to the complainant for her use, in the presence of the aforesaid persons, but the accused have not done the needful of the demand and have thus committed criminal breach of trust punishable u/s 406 IPC."

203

A perusal of the allegations made in the complaint undoubtedly makes out a positive case of the accused having dishonestly misappropriated the articles handed over to them in a fiduciary capacity. To characterise such an entrustment as a joint custody or property given to the husband and the parents is wholly unintelligible to us. All the ingredients of an offense under s.405 IPC were pleaded and a prima facie case for summoning the accused was made out. In such circumstances, the complainant should have been given an opportunity by the High Court to prove her case rather than quashing the complaint. Such an exercise of jurisdiction under s.482 Cr.P.C. is totally unwarranted by law. We might also mention that alongwith the complaint, a list of valuable articles had also been given, the relevant portion of which may be extracted thus;

I. " Jewellery"

1. Nine complete gold sets

2. One complete diamond set

3. Three gold rings

4. Two golden Bahi (Baju Band)

5. One golden chain

6. One shingar patti with golden tikka

7. One golden nath (Nose ring)

8. Twelve golden bangles

II. Silver articles

1. Six glasses and one jug

2. Two surma danies

3. One tagari

4. Two payals

III. Clothes

Fifty one sarees, twenty one suits alongwith petti 204

coats, blouses, nighties, shawls, sweaters, night suits, gowns and woollen coat etc., six complete beds with sheets, etc."

A perusal of the list reveals that so far as the jewellery and clothes, blouses, nighties and gowns are concerned they could be used only by the wife and were her stridhan. By no stretch of imagination could it be said that the ornaments and sarees and other articles mentioned above could also be used by the husband, If, therefore, despite demands these articles were refused to be returned to the wife by the husband and his parents, it amounted to an offence of criminal breach of trust. In mentioning the articles in the list, we have omitted furniture and utensils which though also belonged to the complainant yet there is some room for saying that these were meant for joint use of the husband and wife.

Thus, the facts mentioned in the complaint taken at their face value reveal a clear allegation that the stridhan property of the appellant was entrusted to the husband who refused to return the same to her

Some courts were of the opinion that in view of s. 27 of the Hindu Marriage Act and s. 14 of the Hindu Succession act, the concept of stridhan property of a woman was completely abolished. For instance, the Punjab & Haryana High Court in a case reported in Surindra Mohan etc. v. Smt. Kiran Saini(1) held thus:

"That under the present law on claim can be made on the basis of stridhan, as it has now been completely abolished and cannot avail against statute which makes it the joint property of the parties."

We are of the opinion that this view of the High Court is not legally sustainable because neither of the two Acts, referred to above, go to the extent of providing that the claim of a woman on the basis of stridhan is completely abolished. All that the two sections, mentioned above, provide is that if the husband re-

(1) 1977 Chandigarh Law Report 212

205

fuses to return the stridhan property of his wife, it will be open to the wife to recover the same by properly constituted suit. The sections nowhere provide that the concept of stridhan is abolished or that a remedy under the criminal law for breach of trust is taken away. In a later decision in Bhai Sher Singh & Anr. v. Smt. Virinder Kaur(1), it was very rightly pointed out by the same High Court that s. 27 of the Marriage Act merely provides an alternate remedy to the wife to bring a properly constituted suit in respect of the stridhan property which the husband refused to return. Thus, it is clear that s. 27 merely provides for an alternate remedy and does not touch or affect in any way the Criminal liability of the husband in case it is proved that he has dishonestly misappropriated that stridhan of his wife. It cannot also be spelt out from any textbook or the sastric law of the Hindus that the two Acts mentioned above take away the stridhan right of a woman-at the most these Acts merely modify the concept of stridhan. It may be useful to refer to certain pertinent observations in the aforesaid case.

"The aforementioned passage shows that a female has an absolute right to use her stridhan in any way she likes and even if her husband can take this property at the time of distress, this right is personal to him, The allegations made in the instant complaint are not that the husband of the respondent has placed her ornaments and jewellery etc. Out of her way. What has been alleged therein is that the petitioners who are the parents-in-law of the respondent have converted the ornaments and clothes, etc. presented to the respondent at the time of her marriage to their own use.

Section 27 of the Hindu Marriage Act empowers a Court while deciding a matrimonial dispute to also pass a decree in respect of property which may jointly belong to both the husband and the wife. This section at best provides a civil remedy to an aggrieved wife and does not in any way take away her right to file a crimi-

(1) 1979 Crl. L.J. 493.

206

nal complaint if the property belonging to her is criminally misappropriated by her husband.'' In these circumstances, the decision reported in 1977 Chandigrah Law Reporter 212 can no longer be considered good law. Even in Vinod Kumar's case (supra) the Full Bench reiterated the view that s. 27 in no way abolishes stridhan but expressly recognises the property exclusively owned by the wife. In this connection, the Court observed thus: "The express words of the provision refer to property 'which may belong jointly to both the husband and the wife'. It nowhere says that all the wife's property be longs jointly to the couple or that Stridhan is abolished and she cannot be the exclusive owner thereof. Indeed, in using the above terminology the statute expressly recognises that property which is exclusively owned by the wife is not within the ambit of Section 27 of the Hindu Marriage Act- - -Equally no other provision in the Hindu Marriage Act could be pointed out which erodes the concept of Stridhan or in anyway incapacitates the Hindu wife to hold property as an exclusive owner."

The sheet-anchor of the arguments of the counsel for the respondents-which is based on the decision of the Punjab & Haryana High Court in Vinod Kumar's case-is that the moment a woman after Marriage enters her matrimonial home, her stridhan property becomes a joint properly of both the spouses and the question of application of s. 406 I.P.C is completely eliminated. It is true that to a great extent this part of the argument of the learned counsel is supported by the aforesaid decision but, in our opinion, the decision, so far as this aspect of the matter is concerned, is wholly unsustainable. We would first extract the exact ratio held by the High Court in Vinod Kumar's case: "To conclude, it necessarily follows from the aforesaid discussion that the very concept of the matrimonial home connotes a jointness of possession and custody by the spouses even with regard to the moveable properties exclusively owned by each of them. It is, therefore, inapt to view the same in view of the conjugal relationship as

207

involving any entrustment or passing of dominion over property day-to-day by the husband to the wife or vice versa. Consequently, barring a special written agreement to the contrary, no question of any entrustment or dominion over property would normally arise during coverture or its imminent break-up. Therefore, the very essential prerequisites and the core ingredients of the offence under S.406 of the Penal Code would be lacking in a charge of criminal breach of trust of property by one spouse against the other."

These observations on doubt support the contention of the learned counsel for the respondent but we find it impossible to agree with the aforesaid observations for the reasons that we shall give hereafter. We fail to understand the logic of the reasoning adopted by the High Court in investing the pure and simple stridhan of the wife with the character of a joint property. We are surprised that the High Court should have taken the view that a woman's absolute property though well recognised by law is interpreted by it as being shorn its qualities and attributes once a bride enters her matrimonial home. We are clearly of the opinion that the mere factum of the husband and wife living together does not entitle either of then to commit a breach of criminal law and if one does then he/she will be liable for all the consequences of such breach. Criminal law and matrimonial home are not strangers. Crimes committed in matrimonial home are as much punishable as anywhere else. In the case of stridhan property also, the title of which always remains with the wife though possession of the same may sometimes be with the husband or other members of his family, if the husband or any other member of his family commits such an offence, they will be liable to punishment for the offence of criminal breach of trust under ss. 405 and 406, IPC.

Afterall how could any reasonable person expect a newly married women living in the same house and under the same roof to keep her personal property or belongings like jewellery, clothing, etc., under her own lock and key, thus showing a spirit of distrust to the husband at the very behest. We are surprised how could the High Court permit the husband to cast his covetous eyes on the

208

absolute and personal property of his wife merely because it is kept in his custody, thereby reducing the custody to a legal farce. On the other hand, it seems to that us even if the personal property of the wife is jointly kept, it would be expressly or impliedly kept in the custody of the husband and if he dishonestly misappropriates or refuses to return the same, he is certainly guilty of criminal breach of trust, and there can be no escape from this legal consequence. The observations of the High Court at other places regarding the inapplicability of s. 406 do not appeal to us and are in fact not in consonance with the spirit and trend of the criminal law. There are a large number of cases where criminal law and civil law can run side by side. the two remedies are not mutually exclusive but clearly coextensive and essentially differ in their content and consequence. The object of the criminal law is to punish an offender who commits an offence against a person, property of the State for which the accused, on proof of the offence, is deprived of his liberty and in some cases even his life. This does not, however, affect the civil remedies at all for suing the wrong deer in cases like arson, accidents, etc. It is an anathema to suppose that when a civil remedy is available, a criminal prosecution is completely barred. The two types of actions are quite different in content, scope and import. It is not at all intelligible to us to take the stand that if the husband dishonestly misappropriates the stridhan property of his wife, though kept in his custody, that would bar prosecution under s. 406 I.P.C. Or render the ingredients of s. 405 IPC nugatory or abortive. To say that because the stridhan of a married woman is kept in the custody of her husband, no action against him can be taken as no offence is committed is to override and distort the real intent of the law.

Coming back to the theory of matrimonial home and the stridhan becoming a joint property of the two spouses, the logical effect of the observation made by the High Court is that once a woman enters her matrimonial home she completely loses her exclusive stridhan by the same being treated as a joint property of the spouses. In other words, if this view is taken in its literal sense the consequence would be to deprive the wife of the absolute character and nature of her stridhan and make the husband a co-owner of the same - such a concept is neither contemplated nor known to Hindu law of stridhan, nor does it appeal to pure

209

common sense. It is impossible to uphold the view that once a married woman enters her matrimonial home her stridhan property undergoes a vital change so as to protect the husband from being prosecuted even if he dishonestly misappropriates the same. For instance, properties like jewellery, clothing, cash, etc. given by her parents as gifts cannot be touched by the husband except in very extreme circumstances, viz., where the husband is in imprisonment or is in serious distress. Even then the religion and the law enjoins that the husband must compensate the wife and if he cannot do so, he must pay fine to the King which means that the husband would` be liable to penal action under the present law of the land. - One of the arguments addressed by the counsel for the respondent which had appealed to thee full Bench of the Pun jab & Haryana High Court in Vinod Kumar's case (supra) as also to our learned Brother Varadarajan, J., is that after entering the matrimonial home the custody of the stridhan entrusted by the wife to her husband becomes a sort of a partnership firm and in this view of the matter the question of criminal breach of trust does not arise. In our opinion, it is neither appropriate nor apposite to import the concept of partnership in the relationship of husband and wife for the simple reason that the concept of partnership is entirely different from that of the husband's keeping the stridhan in his custody. Section 4 of the Indian Partnership Act, 1932 (hereinafter referred to as the 'Partnership Act') defines 'partnership' thus:

"partnership" is the relation between persons who have agreed to share the profit of a business carried on by all or any of them acting for all.

Persons who have entered into partnership with one another are called individually "partners" and collectively "a firm" and the name under which their business is carried on is called the "firm name". The essential ingredients of a partnership are: (1) that there should be an actual or physical overt act on the part of two persons to embark an a business adventure.

(2) that if any business is carried on by one or any

210

of the partners the profits of the business shall be shared by - them in the ratio contained in the partnership agreement.

It is, therefore, manifest that in a partnership the wife must by some clear and specific act indicates that the stridhan which has been entrusted to the husband is to be used for a partnership business and the losses of the firm, if any would have to be shared by both. In other words, one of the essential conditions of a partnership firm is that every partner must have dominion over the property by virtue of the fact that he is a partner. This aspect of the matter was highlighted in a decision of this Court in Velji a Raghavji v. State of Maharashtra(1) where the following observations were made:

".. Every partner has dominion over property by reason of the fact that he is a partner. This is a kind of dominion which every owner of property has over his property. But it is not dominion of this kind which satisfies the requirements of s. 405. In order to establish 'entrustment of dominion' over property to an accused person the mere existence of that person's dominion over property is not enough. It must be further shown that his dominion . was the result of entrustment."

In the instant case, however, there is neither any allegation nor anything in the complaint to show that when the wife entered her matrimonial home she had entrusted the property to her husband so as to make him part owner of the same. Therefore, the question of the husband's having dominion over the property does not at all arise. In fact, the wife has nothing to do with the partnership, if any, and the husband is a pure and simple custodian of the property and cannot use the same for any purpose without her consent.A pure and simple act of entrustment of the stridhan to the husband does not attract any of the essential ingredients of a a partnership as defined in the Partnership Act.

In the instant case, there is also no question of the wife constituting herself a partner with her husband merely by allowing him to keep the articles or money in his custody. There is neither any pleading nor any allegation that after her marriage, the appe-

(1)AIR 1965 SC 1433.

211

llant transferred all her properties to her husband for carrying on a partnership business in accordance with the provisions of the Partnership Act. Thus, in our opinion, it cannot be said that a bare act of keeping stridhan property in the custody of the husband constitutes a partnership and, therefore, a criminal case under s. 406 IPC is not maintainable. It is not necessary for us to multiply cases on this point on which there does not appear to be any controversy. We have already pointed out that the stridhan of a woman is her absolute property and the husband has no interest in the same and the entrustment to him is just like something which he wife keeps in a Bank and can withdraw any amount whenever she likes without any hitch or hindrance and the husband cannot use the stridhan for his personal purposes unless he obtains the tacit consent of his wife. When the essential conditions of a partnership do not exist the mere act or factum of entrustment of stridhan would not constitute any co-ownership or legal partnership as defined under s.4 of the Partnership Act.

To sum up the position seems to be that a pure and simple entrustment of stridhan without creating any rights in the husband excepting putting the articles in his possession does not entitle him to use the same to the detriment of his wife without her consent. The husband has no justification for not returning the said articles as and when demanded by the wife nor can he burden her with loss-, of business by using the said property which was never intended by her while entrusting possession of stridhan.) On the allegations in the complaint, the husband is no more and no less than a pure and simple custodian acting on b-half of his wife and if he diverts the entrusted property elsewhere or for different purposes he takes a clear risk of prosecution under s.406 of the IPC. On a parity of reasoning, it is mainfest that the husband, being only a custodian of the stridhan of his wife, cannot be said to be in joint possession thereof and thus acquire a joint interest in the property.

For these reasons, the custody or entrustment of stridhan with the husband does not amount to a partnership in any sense of the term and therefore, we are unable to agree with view taken in Vinod Kumar's case as also with the opinion expressed by our Brother on the points arising in the case.

Another serious consequence as a result of the ratio of the full Bench decision in Vinod Kumar's case would be to render the

212

provisions of s. 406 IPC inapplicable and nugatory even if the husband has the audacity or the importunity of refusing to return the stridhan of his wife. Furthermore, we shall hereafter show that the view of the Full Bench is in direct contravention of a long course of decisions of this Court on the ingredients of s. 405 IPC. Before coming to this chapter, we would like to say a few things more about the judgment of the High Court which on deeper probe and careful scrutiny seems to be self-contradictory.

We are clearly of the opinion that the concept of stridhan property of a married woman becoming a joint property of both the spouses as soon as she enters her matrimonial home and continues to be so until she remains there or even if there is a break in the matrimonial alliance, is in direct contravention of Hindu Law of Sadayika which has been administered since more than a century by High Courts, Privy- Council as also this Court. By a pure and simple figment of the fertile imagination the Judges in Vinod Kumar's case seem to have rewritten the law of criminal breach of trust contained in ss. 405 and 406 IPC so as to carve out an imaginary exception to the application of the Penal Code.A more tragic consequence of the view taken by the High Court is that even if there is a break in the matrimonial alliance and the wife wants her husband to return her exclusive property and he refuses to return, even then the provisions of s. 406 IPC would not apply. It is an extreme travesty of justice for a court to say that whenever a married woman demands- her stridhan property from her husband she should be driven to the dilatory process of a civil court and her husband would be debarred from being prosecuted by a criminal court. By a strange and ingenious process of holding that such an act of a husband does not attract the provisions of the Penal Code, as the property being joint there is no question of the husband being a trustee or holding the same in a fiduciary capacity. Such a view, in our opinion, is not only contradictory but-what the High Court has said before regarding the applicability of s.27 of the Hindu Marriage Act and the nature of stridhan as referred to above-is also neither in consonance with logic and reason nor with the express provisions of the Penal Code and seems to us to be inspired by a spirit of male chauvininism so as to exclude the husband from criminal lability

merely because his wife has refused to live in her matrimonial

213

home. We are indeed surprised how could the High Court, functioning in a civilised and socialistic society such as ours, play havoc with judicial interpretation of an important branch of law.

We shall now show how the final view taken by the High Court is clearly contradictory to what it has observed before. In paragraphs 22A, 23 and 24 of the judgment, the High Court observes as follows:

"It must, therefore, be unreservedly stated that the law, as it stands today, visualises a complete and full ownership of her individual property by a Hindu wife and in this context the factum of marriage is of little or no relevance and she can own and possess property in the same manner as a Hindu male. Once it is held that a Hindu wife can own property in her own right, then it is purely a question of fact whether the dowry or the traditional presents given to her, were to be individually owned by her or had been gifted to the husband alone or jointly to the couple.- .. For instance jewellery meant for the personal wearing of the bride, wedding apparel made to her measures specifically, cash amounts put into a fixed deposit ill a bank expressly in her E name; are obvious examples of dowry raising the strongest, if not conclusive presumption, of her separate owner ship in these articles. Once it is found as a fact that these articles of dowry were so given to her individually and in her own right, then I am unable to see how the mere factum of marriage would alter any such property right and divest her of ownership either totally or partially."

In these paragraphs the High Court unequivocally and categorically expresses the view that a Hindu woman has complete and full ownership of her individual property and the factum of marriage is of no relevance to determine the nature of the property It also holds that articles like jewellery, wedding apparel and cash, etc., cannot alter any such property right. In view of this clear finding given by the High Court, how could it make a complete volte-face by holding that these very properties after marriage become joint property of both the spouses. The High Court has not realised that the theory or philosophy of matrimonial home 214

propounded by it stands directly contradicted by its own observations referred to above.

In paragraph 49 of the judgment, the High Court clearly finds that the mere use by the relations of the husband would not have the effect of passing the possession of the property to the Hindu undivided family and in this connection observes thus :-

Equally, the common use and enjoyment of certain articles of dowry and traditional presents, by the other members of a joint family with the leave and licence of a Hindu wife, cannot have the effect of extending the jointness Of control and custody of the couple to undefined and unreasonable limits. Consequently, there is no reason to assume that the mere user or enjoyment of the dowry by other members of the house-hold, would have the effect of passing the possession and control thereof jointly to the Hindu Undivided Family as such."

Thus, these observations run counter and are totally inconsistent and irreconcilable with the view taken by the High Court in paragraph 41 where it has observed thus: "In the light of the above it would be farcical to assume that despite the factum of a marriage and a common matrimonial home the two spouses would stand in a kind of a formal relationship where each is entrusted with or has been passed dominion over the exclusive property of the other..-....The matrimonial home so long as it subsist presumes a jointness of custody and possession by the spouses of their individual as also of their joint properties line.. The inevitable presumption during the existence or the imminent break up of the matrimonial home there fore is one of joint possession of the spouses which might perhaps be dislodged by the special terms of a written contract. However, to be precise this presumption of joint possession properties within the matrimonial home can subsist only as long- as the matrimonial home subsists or on the immediate break up thereof."

At other places the High Court has observed thus: 215

"47. In view of the above, it would be equally untenable to hold that either the desertion or the expulsion of one of the spouses from the matrimonial home would result in entrusting dominion over the property belonging to the other so as to bring the case within the ambit of this pre requisite under S.405, Indian Penal Code. The joint custody and possession once established would thereafter........ exclude either express entrustment or the passing of dominion over the property. It was rightly argued that if an irate husband or wife walks out from the matrimonial home in a huff, this cannot constitute an entrustment or dominion over the property to the other. Consequently, unless a special written agreement to the contrary can be established, the strongest presumption arises that during the existence and immediately after the crumbling of the matrimonial home, there was in essence, a joint possession and custody of the property of the spouses therein, including dowry and traditional presents, which would preclude the essentials of entrustment or dominion over the property which form the cornerstone of criminality under s.405, Indian Penal Code.

53. It cannot, therefore, be prim. facie presumed that these are exclusively the ownership of the wife or inevitably entrusted either to the husband or his close relations. As was noticed earlier, if an irate wife in a tantrums abandons the matrimonial home, such like property does not in the eye of law become entrusted to the parents-in-law or other close relations of the husband No such gullible presumption of entrustment or passing of the dominion of property can be raised in such a situation to come within the mischief of criminality for breach of trust. Entrustment or dominion over the property has to be unequivocally alleged and conclusively established by proof later." The High Court had itself rightly spelt out the legal propositions that the pure and traditional presents given to a bride in a Hindu wedding may be divided into three categories, viz.,

(a) property intended for exclusive use of the bride, e. g., her personal jewellery, wearing apparel, etc.

216

(b) articles of dowry which may be for common use and enjoyment in the matrimonial home, and

(c) articles given as presents to the husband or the parent-in -law and other members of his family. With regard to category (a) above, the High Court observed thus:

"Similarly as regards the first category of articles meant for the exclusive use of the bride she would retain her pristine ownership therein irrespective or her entry and presence in the matrimonial home or that of her parents in-law." The High Court thus accepts the well established rule of Hindu law of stridhan that that articles mentioned in category (a) are meant for the exclusive use of the bride and are her personal property. Unfortunately, however, with regard to category (c) while discussing the question of the rights of the bride to her exclusive property upon her entry in her matrimonial home, the High Court has wrongly applied what it had previously held with regard to category (a). In one breath the Judges say that the bride is entitled to retain her ownership irrespective of her entry and presence in the matrimonial home and in the other they come to the conclusion that the moment a married woman enters her matrimonial home, all her properties, including her exclusive property, become a joint property by a fiction of being placed in the custody of her husband or his relations. While we agree with the first part of the categories, as extracted above, we find it difficult to accept the other propositions adumbrated at a later stage of the judgment which have been fully discussed by us. We fail to understand how the High Court while finding that joint enjoyment does not divest a Hindu wife of her exclusive ownership still chose to treat it a joint property of the two spouses by the mere factum of joint user. The two views expressed by the High Court stand contradicted by its own findings and are wholly understandable. Thus, a detailed analysis of the judgment of the Punjab & Haryana High Court in Vinod Kumar's case (supra) appears to us to be a mass of confusion and lacks both clarity and coherence. We are, therefore, unable 217

to uphold or support the view of the High Court that upon entering the matrimonial home the ownership of stridhan property becomes joint with her husband or his relations- To this extent, therefore, we overrule this decision and hold that with regard to the stridhan property of a married woman, even if it is placed in the custody of her husband or in-laws they would be deemed to be trustees and bound to return the same if and when demanded by her. The Supreme Court in a large number of cases has held that the fundamental core of the offence of criminal breach of trust. is that a property must be entrusted and the dominion of the property should be given to the trustee. In the present case, all these conditions, even according to the findings of the Court though not its conclusion, are clearly established. That the view of the High Court is absolutely wrong would be clear from a number of authorities, some of which we would like to discuss here. In Chelloor Manaklal Narayan Ittiravi Nambudiri v. State of Travancore(1) this Court made the following observations:

"As laid down in S. 385, Cochin Penal Code (corresponding to S. 405, Indian Penal Code) to constitute an offence of criminal breach of trust it is essential that the prosecution must prove first of all that the accused was entrusted with some property or with any dominion or power over it-It follows almost axiomatically from this definition that the ownership or beneficial interest in the property in respect of which criminal breach of trust is alleged to have been committed, must be in some person other than the accused and the latter must hold it on account of some person or in some way for his benefit."

In Jaswantrai Manilal Akhaney v., State of Bombay(2) Sinha, J. (as he then was) observed thus: "For an offence under section 409, Indian Penal Code, the first essential ingredient to be proved is that the property was entrusted- - But when section 405 which defines "criminal breach of trust speaks of a person being

(1) AIR 1953 SC 478.

(2) [1956] S.C.R. 483.

218

in any manner entrusted with property, it does not contemplate the creation of a trust with all the technicalities of trust. It contemplates the creation of a relationship whereby the owner of property makes it over to another person to be retained by him until a certain contingency arises or to be disposed of by him on the happening of a certain events."

In Akharbhai Nazorali v. Md. Hussain Bhai(1) the Madhya Pradesh High Court made the following observations: "It may be that the deduction and retention of the employees' contribution is a trust created by virtue of that very fact, or by virtue of a provision in statute or statutory rule. But even apart from the latter, the mere fact of telling the employees that it is their contribution to the provident fund scheme and then making a deduction or recovery and retaining it, constitutes the offence of criminal breach of trust. This is so obvious that nothing more need be said about it."

These observations were fully endorsed and approved by this Court in Harihar Prasad Dubey v. Tulsi Das Mundhra & Ors.(2 where the following observations were made: "This, in our opinion, is a correct statement of the position and we also agree with the learned Judge of the Madhya Pradesh High Court that "this so obvious that nothing more need be said about it We, therefore, think that the impugned order quashing the charge against the respondents is obviously wrong." In Basudeb Patra v. Kanai Lal Haldar(3) the Calcutta High Court observed thus:

"Whereas the illustration to s. 405 show equally clearly that the property comes into.

(1) AIR 1961 M. P. 37:

(2) AIR 1981 SC 92.

(3) AIR 1949 Calcutta 207,

219

the possession of the accused either by an express entrustment or by some process placing the accused in a position of trust..-.-On the facts of the present case, which, as I have said, are not open to question at this stage, it is quite clear that the ornaments were handed over to the petitioner by the beneficial owner in the confidence that they would be returned to the beneficial owner in due time after having been used for the purpose for which they were handed over. If this is not an entrustment, if is impossible to conceive what can be an entrustment."

(Emphasis ours)

This ratio was fully approved by this Court in Velji Raghavji Patel v. State of Maharashtra(1) where the following observation were made:

"In order to establish " entrustment of dominion" over property to an accused person the mere existence of that person's dominion over property is not enough. It must be further shown that his dominion was the result of entrustment. Therefore, as rightly pointed out by Harris, C.J. the prosecution must establish that dominion over the assets or a particular asset of the partnership was by a special agreement between the parties, entrusted to the accused person." In the case of State of Gujrat v. Jaswantlal Nathalal,(2) Hegde, J., speaking for the Court, observed thus:

"The expression 'entrustment' carries with it the implication that the person handing over any property or on whose behalf that property is handed over to another, continues to be its owner. Further the person handing over the property must have confidence in the person taking the property so as to create a fiduciary relationship between them."

In Sushil Kumar Gupta v. Joy Shanker Bhattacharjee(3) this Court observed thus:

(1) AIR 1965 SC 1433.

(2) [1968] 2 SCR 408.

(3) AIR 1971 SC 1543.

220

"The offence of criminal breach of trust is committed when a person who is entrusted in any manner with property or with dominion over it, dishonestly misappropriates it or converts it to his own use.. The appellant's manner of dealing with the money entrusted to his custody clearly constitutes criminal breach of trust."

In the case of Superintendent & Remembrancer of Legal Affairs, West Bengal v. S.K. Roy (1) this Court held that for 'entrustment' two things are necessary, viz., (l) the entrustment may arise in "any manner" whether or not it is fraudulent, and (2) the accused must have acquisition or dominion over the property.

In Bhai Sher Jang Singh & Anr. v. Smt. Virinder Kaur (supra) the Punjab & Haryana High Court observed thus: It might be that some of the articles which were presented to her are for the use of both the spouses but the ornaments and things of the like nature are certainly meant for her and her alone. When she makes an allegation in the complaint that either her husband or her parents-in-law had converted to their own use the ornaments forming the part of her stridhan which she had entrusted to them, the Court has to give legal effect to such allegation and to assume that such ornaments had been made the subject matter of criminal breach of trust. It is settled law that even in a criminal complaint the complainant is under no obligation to plead the legal effect of the allegations made. All that is required is that the facts constituting a complaint should be specifically mentioned so that the Court may be able to perform its duty of punishing the accused under the appropriate provision of law if such allegations are made out. Further more, in a case like this a complaint cannot be quashed without giving the aggrieve wife an opportunity of proving that the ornaments had been given to her at the time of her marriage for her use only." (Emphasis supplied)

We fully endorse this decision and hold that it lays down the correct law on the subject.

(1) [1974] 4 S.C.C. 230.

221

There is a judgment of the Allahabad High Court which more or less takes the same view as the Punjab & Haryana High Court in Vinod Kumar's case (supra). In Criminal Misc. Case No. 676 of 1981 (connected with) Criminal Misc. Case No. 2753 of 1981, Kailash Nath Agarwal & Ors. v. Prem Pal Agarwal & Anr., (decided on 22.12.1983), the Allahabad High Court, out of the three categories laid down by Punjab & Haryana High Court in Vinod Kumar's case, accepted only the third category, viz., articles which constitute the individual property of the person for whose use it was given, and held that the rest of the property falling under categories (a) and (b) would be property exclusively meant for the use of the bride and once it was brought to the family home, the possession would be joint unless by an express written agreement there was an entrustment of the property of the bride to other members of the family. The Allahabad High Court thus also accepts the concept of the property being a joint property in the matrimonial home. By and large this decision toes the line of the view taken by the Punjab and Haryana High Court in Vinod Kumar's case. Furthermore, the High Court has gravely erred in holding that the property could only be claimed by filing a properly constituted civil suit or in accordance with the provisions of the Dowry Prohibition Act or the Hindu Marriage Act as the case may be. This proposition, in our opinion, is wholly incorrect as conceded even by the Punjab , & Haryana High Court in Vinod Kumar's case.

There is an earlier decision of the Punjab & Haryana High Court which clearly holds that where there is a clear allegation of entrustment by the wife against the husband, he could be prosecuted by a criminal court on a complaint filed by the wife. In this connection, the Court in Avtar Singh & Anr. v. Kirpal Kaur Criminal Misc. No.2144-M of 1979 and Criminal Misc.No.2145 of 1979, decided on 16 8.79) made the following observations:

"In my opinion, where certain thing is lying in trust with a person, offence of dishonest misappropriation would be committed on a date the demand for return of the entrusted articles is made and the same is declined...According to the complaint, the first demand for the return of the articles was made on January 27, 1976 and it was that date when the demand was declined. Hence, the offence of misappropriation of the dowry articles lying in trust was committed on January 27, 1976."

222

We find ourselves in entire agreement with this decision and hold that this was correctly decided. This Court has pointed out more than once that the High Court should very sparingly exercise its discretion under s. 482 Cr. P.C. In L.V. Jadhav v. Shankarrao Abasaheb Pawar & Ors.(l) (to which two of us were a party), this Court made the following observations:

"The High Court, we cannot refrain from observing, might well have refused to invoke its inherent powers at the very threshold in order to quash the proceedings, for these powers are meant to be exercised sparingly and with circumspection when there is reason to believe that the process of law is being misused to harass a citizen."

In Smt. Nagawwa v. Veeranna Shivalingappa Konjalgi & Ors.(2) this Court observed as follows :- "Thus, it may be safely held that in the following cases an order of the magistrate issuing process against the accused can be quashed or set aside: (1) Where the allegations made in the complaint or the statements of the witnesses recorded in support of the same taken at their face value make out absolutely no case against the accused or the complaint does not disclose the essential ingredients of an offence which is alleged against the accused;

(2) Where the allegations made in the complaint are patently absurd and inherently improbable so that no prudent person can ever reach a conclusion that there is sufficient ground for proceeding against the accused;

(3) where the discretion exercised by the Magistrate in suing process is capricious and arbitrary having been either on no evidence or on materials which are wholly irrelevant or inadmissible; and (1) AIR [1983]SC 1219.

(2) [1976] Supp. SCR123

223

(4) where the complaint suffers from fundamental legal defects, such as, want of section, or absence of a complaint by legally competent authority and the like.

The cases mentioned by us are purely illustrative and pro vide sufficient guidelines to indicate contingencies where the High Court can quash proceedings." B

The same principles would apply mutatis mutandis to a criminal complaint.

We now come to the question as to whether or not a clear allegation of entrustment and misappropriation of properties was made by the appellant in her complaint and, if so, was the High Court justified in quashing the complaint at that stage. It is well settled by a long course of this Court that for the purpose of exercising its power under s. 482 Cr.P.C. to quash a FIR or a complaint the High Court would have to proceed entirely on the basis of the allegations made in the complaint or the documents accompanying the same per se. It has no jurisdiction to examine the correctness or otherwise of the allegations. In case no offence is committed on the allegation and the ingredients of s.405 & 406, I.P.C. are not made out, the High Court would be justified in quashing the proceedings. In the present case, we shall show that the allegations are both clear, specific and unambiguous and, therefore, the complainant should have been given a chance to prove her case. It is, of course, open to the accused at the trial to take whatever defence that were open to him but that stage had not yet come and therefore, the High Court was totally ill-advised to speculate on the merits of the case at that stage and quash the proceedings. We have narrated the facts in detail in the earlier part of our judgment but we might again, even at the risk of repetition, indicate the bare facts which prima facie make out a clear case under s.406, IPC against the accused. The important portions of the complaint may be spelt out thus:

(1) that all the accused attended the marriage of the appellant with the respondent and demanded dowry from the parents of the appellant in consideration of the marriage.

(2) that the parents of the appellant spent Rs,75,000 on the marriage and dowry articles worth Rs.60,000 224

(inclusive of jewellery, wearing apparel, etc.) were given and entrusted to accused Nos.1 to 6 at the time of the Doli on 5.2.72,

(3) that the articles entrusted to the accused were meant for the exclusive use of the appellant, (4) that the dowry articles were never given by the accused to the appellant even for her use and possession of the same was illegally, dishonestly and mala fidely retained by the accused in order to obtain a wrongful gain to themselves and wrongful loss to the appellant,

(5) that on 11.12.1980 in the morning, the accused brought the appellant to Ludhiana in three clothes and refused to give the entrusted articles which were the stridhan of the appellant.

Taking all the allegations made above, by no stretch of imagination can it be said that the allegations do not prima facie amount to an offence of criminal breach of trust against the respondent. Thus, there can be no room for doubt that all the facts stated in the complaint constitute an offence under s. 406 IPC and the appellant cannot be denied the right to prove her case at the trial by per-empting it at the very behest by the order passed by the High Court. We therefore, overrule the decisions of the Punjab & Haryana High Court in Vinod Kumar's case. By way of post-scriptt we might add that we are indeed amazed to find that so deeply drowned and inherently engrossed are some of the High Courts in the concept of matrimonial home qua the stridhan property of a married women that they simply refuse to believe that such properties are meant for the exclusive use of the wife and could also be legally entrusted to the husband or his relatives. Thus, if the husband or his relatives misappropriate the same and refuse to hand it over to the wife and convert them to their own use and even though these facts are clearly alleged in a complaint for an offence under s. 405/406 I.P.C., some courts take the complaint is not maintainable. Thus, even when clear and specific allegations are made in the complaint

225

that such properties were entrusted to the husband, they refuse to believe these hard facts and brush them aside on the ground that they are vague. The allegations of the complainant in this appeal and the appeal before the Allahabad and the Punjab & Haryana High Court show that it is not so but is a pure figment of the High Court's imagination as a result of which the High Court completely shut their eyes to the fact that the husband could also be guilty under s. 405/406 I P.C. in view of the clear allegations made in the complaint. In other words, the High Courts simply refuse to believe that there can be any such entrustment and even if it is so no offence is committed. Such an approach amounts to a serious distortion of the criminal law, resulting in perpetrating grave and substantial miscarriage of justice to the wife at the hands of the High Courts. We cannot countenance such a wrong and perverse approach.

For the reasons given above, we are satisfied that as the complaint prima facie disclosed an offence of criminal breach of trust as defined in s. 405/406 of the Indian Penal Code the High Court was not justified in quashing`the complaint. We, therefore, allow this appeal, set aside the judgment of the High Court and restore the complaint filed by the appellant and direct that the accused may be summoned, if not already summoned, and put on trial in accordance with law.

VARADARAJAN, J. This criminal appeal by special leave is directed against the judgment of a learned Single Judge of the Punjab and Haryana High Court in Criminal Misc. Case No.4876 of 1981.

The appellant, Pratibha Rani is the estranged wife of the first respondent Suraj Kumar who is the brother of the second respondent Krishan Lal. One Rattan Chand is the father of respondents 1 and 2 and two others Chander Kumar and Vishwinder Kumar. One Jugal Kumar is the brother-in-law of the first respondent.

The appellant filed a criminal complaint for an offence under s.406 I.P.C. against her husband and his father and brothers and brother-in-law mentioned above in the Court of the Additional Chief Judicial Magistrate, Ludhiana, alleging that she was married to the first respondent at Ludhiana on

4. 2. 1972 according to the Hindu rites and customs. The material averments in the complaint

226

are these: The aforesaid persons, namely, father, brother and brother-in-law of the first respondent attended the marriage and demanded dowry from the appellant's parents as consideration for the marriage. Accordingly, dowry articles mentioned in the list appended to the complaint, worth Rs. 60,000, in the form of golden articles, clothes and other valuables were given and entrusted to the respondents and four others mentioned in the complaint at Ludhiana time of 'doli' on 5.2.1972 in the presence of Kapur Chand Jain and six others. The six respondents in the complaint started teasing, harassing and beating the appellant and they kept her without even food to extract more money from her parents. They turned out the appellant with her children in the beginning of 1977. After a great deal of persuasion and intervention by Panchayatdars, respondent 1 came to Ludhiana and took the appellant to his house after giving an undertaking in writing on 21. 6. 1977 not to misbehave with and maltreat the appellant her children. But after some time all the respondents in the complaint started maltreating the appellant and misbehaving with her. The articles mentioned in the list were never given by the respondents in the complaint to the appellant for her use but were retained by them illegally and with the dishonest intention of causing wrongful gain to themselves and wrongful loss to the appellant. The respondents in the complaint brought the appellant to Ludhiana at 4.30 a.m. On 11.12.1980 and left her near Kailash Cinema Chowk. They refused to give the articles mentioned in the list which are the stridhan of the appellant to her. When the appellant's husband and his brother, Vishwinder Kumar, respondents 1 and 5 in the complaint, came to Ludhiana on 10.2.1981 to attend the proceeding started by the appellant under s. 125 Cr. P.C., her parents persuaded them to return the articles entrusted to them at the time of the marriage but they flatly refused to comply with that demand. The articles have not been returned in spite of service of notice dated 17. 12. 1981 on the first respondent. Thus the respondents in the complaint have dishonestly converted the articles belonging to the appellant for their use in violation of the direction of the appellant's parents given at the time of the marriage to give the articles for the appellant's use. The respondents in this appeal filed Criminal Misc. Case No.4876 of 1981 in the Punjab and Haryana High Court under s.482 of the Code of Criminal procedure for quashing the criminal

227

Proceedings and the complaint taken on file by the Additional Chief Judicial Magistrate, Ludhiana under s. 406 I.P.C. and his order summoning them.

Sukhdev Singh Kang, J. before whom the matter came up in the High Court relied strongly upon the observations made by a Full Bench of that High Court in Vinod Kumar Sethi & Ors. v. State of PunJab and Ors.(l) and has observed in his judgment that the mere handing over of the articles of dowry of stridhana to the husband and other relations at the time of the marriage does not constitute entrustment in the sense of the word used in ss. 405 and 406 I P.C. and that it does not amount to passing of dominion over those articles to them. The learned Judge has observed that there can be such an entrustment only by a subsequent conscious act of volition ` and that in the absence of such an act any allegations of breach of trust between the husband and wife cannot constitute an offence under s.406 I.P.C. The learned Judge has further observed that between the husband and wife there is always a jointness of control and possession of the properties of the spouse within the matrimonial home and that it goes against the very concept of entrustment of his or her property by one spouse to the other. In this view, he allowed the petition and quashed the proceeding arising out of the appellant's complaint, observing that the allegations in the appellant's complaint are similar to the one in Vinod Kumar's case (supra) and that this case is fully covered by the ratio in that decision.

The appellant has, therefore, come to this Court in appeal by special leave, impleading the petitioners before the High Court, who are only two out of the six respondents in the complaint, as respondents in this appeal. In a petition under s.482 Cr.P.C. for quashing a criminal complaint, the allegations made in the complaint have to be taken to be correct in order to find out whether they constitute the various ingredient of the offence alleged. In Nagawa Veernna Shivalingappa Konjalgi & Ors ) illustrations have been given of cases in which it may be safely held that an order of a Magistrate issuing process against an accused can be quashed or set aside. They are: (1) AIR 1982 Punjab 372.

(2) [1976] Suppl S.C.R. 123

228

(1) Where the allegations made in the complaint or the statements of the witnesses recorded in support of the same, taken at their face value, make out absolutely no case against the accused or the complaint does not disclose the essential ingredients of an offence which is alleged against the accused; (2) Where the allegations made in the complaint are palpably absurd and inherently improbable so that no prudent person can ever reach a conclusion that there is sufficient ground for proceeding against the accused;

(3) Where the discretion exercised by the Magistrate in issuing process is capricious and arbitrary having been based either on no evidence or on materials which are wholly irrelevant or inadmissible; and

(4) Where the complaint suffers from fundamental legal defects such as want of sanction, or absence of a complaint by a legally competent authority and the like."

Article 126 in Mulla's Hindu Law, Fifteenth Edition, describing what constitutes Stridhana reads: - "property given or bequeathed to a Hindu female whether during maidenhood, coverture or widowhood by her parents and their relation or by her husband and his relations is stridhana according to all schools except that the Dayabhaga does not recognise immovable property given or bequeathed by husband to his wife as stridhana."

Section 2 of the Dowry prohibition Act, 1961 defines "dowry" as meaning:

"any property or valuable security given or agreed to be given either directly or indirectly-(a) by one party to a marriage to the other party to the marriage, or (b) by the parents of either party to the marriage, or by any other person to either party to the marriage or to any other person at or before of after the marriage in connection with the marriage of the said parties but does not include dower or mahr in the case of person to whom the Muslim personal law (Shariat) applies."

229

In the present complaint of the wife against the husband and , his three brothers, father and brother-in-law, it is alleged that the marriage was performed at Ludhiana on 4.2.1972 according to Hindu rites and customs and that the father and three brothers and the brother-in law of the husband attended the marriage and demanded dowry from the wife's parents as consideration for the marriage and that accordingly dowry articles worth Rs.60,000, mentioned in the list attached to the complaint, consisting of gold articles, clothes and other valuables were given and entrusted to the husband and the other five respondents in the complaint, at the time of the 'doli' at Ludhiana on 5.2. 1972 in the presence of Kapur Chand Jain and six other persons. For the purpose of the petition under s.482 Cr.P.C. those articles must be prima facie considered to be dowry or stridhana of the appellant-wife.

In Velji Raghavjl Patel v. State of Maharashtra,(1) it is observed:

"Upon the plain reading of s.405, I.P.C. it is obvious that before a person can be said to have committed criminal breach of trust it must be established that he was either entrusted with or entrusted with dominion over property which he is said to have converted to his own use or disposed of in violation of any direction of law etc. Every partner has dominion over property by reason of the fact that he is a partner. This is a kind of dominion which every owner of property has over his property. But it is not dominion of the kind which satisfies the requirements of s. 405. In order to establish "entrustment of dominion" over property to an accused person the mere existence of that person's dominion over property is not enough. It must be further shown that his dominion was the result of entrustment. Therefore, as rightly pointed out by Harris C.J., the prosecution must establish that dominion over the assets or a particular asset of the partnership was by a special agreement between the parties, entrusted to the accused person. If in the absence of such a a special agreement a partner receives money belonging to the partnership he cannot be said to have received it in a

(1) [1965] 2 S C.R. 429

230

fiduciary capacity or in other words cannot be held to have been "entrusted" with dominion over partnership properties."

In State of Gujarat v. Jaswantlal Nathalal(1) it is observed:

"Before there can be any entrustment there must be a trust meaning thereby an obligation annexed to the owner ship of property and a confidence reposed in and accepted by the owner or declared and accepted by him for the benefit of another or of another and the owner. But that does not mean that such an entrustment need conform to all the technicalities of the law of trust - seeJaswantrai Manilal Akhaney v. State of Bombay [1956] SCR 483, 498-500. The expression 'entrustment' carries with it the implication that the person handing over any property or on whose behalf that property is handed over to anther, continues to be its owner. Further the person handing over the property must have confidence in the person taking the property so as to create a fiduciary relationship between them." In Sushil Kumar Gupta v. Joy Shankar Bhattacharyya(2), it is observed:

"The offence of criminal breach of trust is committed when a person who is entrusted in any manner with property or with dominion over it, dishonestly misappropriates it, or converts it to his own use, or dishonestly uses it or disposes it of, in violation of any direction of law prescribing the mode in which the trust is to be discharged, or of any lawful contract, express or implied, made by him touching such discharge, or wilfully suffers any other person so to do."

In Superintendent Remembrancer of Legal Affairs, West Bengal v. S.K. Roy(8), it is observed:

"There are, however, two distinct parts involved in the commission of the offence of criminal breach of trust. The first consists of the creation of an obligation in rela

(1) [1968] 2 SCR 408.

(2) [1970] 3 SCR, 770.

(3) [1974] 4 SCC,230.

231

tion to the property over which dominion or control is acquired by the accused. The second is a misappropriation or dealing with the property dishonestly and contrary to the terms of the obligation created.

The most important ingredient of an offence under s. 406, which is alleged by the wife against her husband, his three brothers, father and brother-in-law in her complaint in the present case is the entrustment of the dowry articles to the respondent in the complaint and ,their dishonest conversion thereof to their own use. There is no doubt an allegation in the complaint that these articles were given and entrusted to the respondents in the complaint at Ludhiana at the time of doll on 5-2-1972. Apart from the husband the other respondents in the complaint, as already stated, are his father, three brothers and brother-in-law. The articles were given for the use of the wife- If so, could there be entrustment of the articles to such a number of diverse persons? In the background of what usually happens in Hindu marriages namely, placing of the articles presented to the bride in the presence of the elders and others assembled for the occasion and removal thereof after the function is over it has to be seen whether the allegation made in the complaint amounts to entrustment as required by law to make out an offence under s. 406 l.P.C. This question has been considered in detail by a Full Bench of the Punjab and Haryana High Court in Vinod Kumar's case (supra) after an analysis of several decision relating to the question. The learned Single Judge who has quashed the complaint in the present case on a petition of the husband and one of his brothers has heavily relied upon that Full Bench decision of his Court. What runs through the judgment of the learned Judges in that case is the concern of the Court for the peaceful and harmonious relationship between the spouses in a matrimonial home and a careful consideration of the question whether the ingredient of entrustment" exists in such cases. Therefore, it is necessary to note what has been observed in some of the paragraphs of the judgment to that case. The learned Chief justice speaking for the Bench has observed: "21.. The present set of cases presents a sad spectacle of a house divided against itself, not merely in the biblical but in the literal sense, where wives are ranged against their husbands in acrimonious criminal prosecu-

232

tions. The challenge on behalf of the husbands and their relations is focussed basically against the charge of breach of trust under Section 406 of the Indian Penal Code, levelled against them. Now the core of the argument on behalf of the petitioners is that the very concept of any entrustment or passing dominion over her property by the wife to the husband does not arise at all so long as the marriage subsists. The contention is that the very nature of the conjugal relationship itself would negative any such stand. On this premise it is contended that the basic pre- requisite of the entrustment of property or dominion over property being lacking and non-existent, no offence under Section 406, Indian renal Code, can possibly be made out. Therefore, it was argued that even accepting the first information reports as they do not and indeed cannot disclose a cognizable offence under Section 406. The petitioners, therefore, seek the quashing of the proceedings - forthwith rather than being obliged to go through the tortuous mill of a police investigation or the consequent criminal trial." "25. Now apart from the principle, the most ancient texts of Hindu Law have always been categoric that dowry, as commonly understood, was stridhana and thus in the exclusive ownership of the bride." "26. Now once it is so held that articles of dowry and traditional presents given at the wedding are owned by the bride individually in her own right, then one fails to see how by the mere fact of her bringing the same into her husband's or parents-in-law's household, would forth with divest her of the ownership thereof. Separate and individual right to property of the wife therein cannot vanish into thin air the moment the threshold of the matrimonial home is crossed. To say that at that point - of time she would cease to own such property altogether and the title therein would pass to her husband or in any case she would lose half of her right therein and become merely a joint owner of the same, with the family of her husband, does not appear to me as even remotely warranted either by the statute, principles or logic. No such marriage hazard against the wife can be implied in law.

233

Once she owns property exclusively, she would continue to hold and own it as such despite marriage and coverture and the factum of entering the matrimonial home.. "

"35. To conclude on this aspect, I find nothing in the codification of Hindu Law which in any way abolishes the concept of stridhana or the right of a Hindu wife to exclusive individual ownership. Indeed the resultant effect of such enactments is to put the Hindu female wholly at par with the Hindu male, if not at a higher pedestal with regard to individual ownership of the property."

40. Now having held as above that Hindu wife can exclusively own and hold property including her dowry and traditional presents given at the wedding, the decks are cleared for tackling the core question posed at the very outset. What indeed is the true legal relationship of the husband and wife qua the property individually owned by each within the four walls of the matrimonial home? Does the wife stand entrusted with the property belonging to her husband individually and vice versa the husband stands entrusted with such property vesting in the exclusive ownership of the wife? It is the answer to this question which in essence would determine the attraction and applicability of Section 405, I.P.C betwixt the spouses.."

"41. It bears 'repetition that the question herein has to be examined against the backdrop of the matrimonial home. What truly is the concept and essence thereof had come up for exhaustive consideration earlier before a Full Bench in Kailash Vati v. Ayodhia Parkash, ILR (1977) 1 Punj. & Har. 642 in the context of Hindu Law itself. It is, therefore, apt to refer to the authoritative enunciation therein:-

"To my mind, the idea of the matrimonial home appears to lie at the very centre of the concept of marriage in all civilised societies. It is indeed around it that generally the marriage tie revolves. The home epitomizes the finer nuances of the marital status. The bundle of indefinable rights and duties which

234

bind the husband and the wife can perhaps be best understood only in the context of their living together in the marital home The significance of the conjugal home in the marriage tie is indeed so patent that it would perhaps be wasteful to elaborate the 8 same at any great length. Indeed, the marital status and the conjugal home have been almost used as interchangeable terms." and

"To summarise, I have attempted to show by reference to Anglo-American Jurisprudence that the a concept of the marital home lies at the very centre of the idea of marriage in all civilised societies. Perhaps from primeval times when human beings lived sheltered in subterranean caves to the modern day when many live perched in flats in high rise apartments within the megapolis, the husband and the wife have always hankered for a place which may be their very own and which they may call a home. The innumerable mutual obligations and rights which stem from the living together of man and wife are undoubtedly beyond any precise definition and stand epitomized by the concept of the matrimonial home."

In the light of the above it would be farcical to assume that despite the factum of a marriage and a common matrimonial home the two Spouses would stand in a kind of a formal relationship where each is entrusted with or has been passed dominion over the exclusive property of the other. Rather it appears to me that the conjugal relationship and the existence of a matrimonial home automatically obviates any such hyper-technicalities of an entrustment or dominion over property. It seems inapt to conceive the relationship as a day-to-day entrustment of the property of the husband to the custody of the wife or vice versa of the property of the wife to the husband. The matrimonial home so long as it subsists presumes a jointness of custody and possession by the spouses of their individual as also of their joint properties which can not be divided by any metaphorical line. In a homely metaphor in the context of the modern commercialised world it has been said that the marriage relationship is not one of

235

"I and You limited" but that of "We limited". Whilst the law undoubtedly now clearly recognises the individual ownership of property by the husband and wife, the necessary assumption in law, therefore, would be that during the existence or even the imminent break up the matrimonial home the concept of jaintness of possession therein seems to be a paramount one. The inevitable presumption during the existence or the imminent break up of the matrimonial home therefore is one of joint possession of the spouses which might perhaps be dislodged by the special terms of a written contract. However, to be precise this presumption of joint possession of properties within the matrimonial home can subsist only as long as the matrimonial home subsists or on the immediate break up thereof." "42-43. The aforesaid position seems to be well borne out by a homely example which was rightly advanced by Mr. Bhandare on behalf of the petitioners. It was submitted that where a husband entrusts a specific amount to a wife for paying the school fees of their children but in a shopping spree she converts the same into sarees for herself, would she thereby become liable to breach of trust under Section 406, Indian Penal Code? The answer would obviously appear to be in the negative. Similarly where a husband misuses or even appropriates any property exclusively belonging to his wife within the matrimonial home he hardly comes within the ambit of criminality under Section 406, Indian Penal Code. Usually if not invariably where the husband is the bread winner he brings home the month's wages and bands them over to the wife to be spent on the family. Would it be possible to say that if she use the same for herself and even against the consent of her husband she would be committing a criminal breach of trust? Obviously the answer would appear to be in the negative."

"44. One may now turn precisely to the language of the Code itself. Sec. 405 is in the following terms:- 236

"405. Criminal Breach of trust: Whoever being in any manner entrusted with property, or with any dominion over property, dishonestly

misappropriates or converts to his own use that property, or dishonestly uses or disposes of that property in violation of any direction of law prescribing the mode in, which such trust is to be discharged or of any legal contract, express or implied, which he has made touching the discharge of such trust, or wilfully suffers any other Person so to do, commits criminal breach of trust."

It is well-setted that from a legal contract, or violation of direction of law, the entrustment of property or dominion over property are the per- requisites for the applicability of the aforesaid provision. Once it is held as above, that property within the matrimonial home is in the joint possession and custody (despite rights of the individual ownership therein) then these very per-requisites of entrustment or dominion over property cannot be easily satisfied betwixt the spouses inter se. It is indeed well-settled that the very concept of the jointness of possession and custody would rule out the entrustment or dominion over property betwixt such joint custodians. In line with the concept of joint ownership where the possession of one joint owner is deemed to be the possession of all, the analogy,is to be extended that existence of the property within the matrimonial home rises a presumption that both the husband the wife are in possession thereof jointly and not that each one has entrusted his exclusive property to the custody of other. Subscribing to the latter view would be both overly hypertechnical and subversive of the very concept of marriage, the matrimonial home and the inevitable mutual trust which conjugality necessarily involves."

"45. It is obviously because of the afore said legal position and this inarticulate peremise underlying the same that the learned counsel for the State and the complainants were unable to cite even a single case of conviction for criminal breach of trust betwixt husband and wife. Even when

237

pointedly asked, counsel conceded that despite the diligent research neither under the Indian Penal Code, nor under the analogous provisions of English law could they lay their hands for over a century and a half on any case where such a conviction had been upheld. This paucity, rather the total absence of precedent, indirectly buttresses the view I have expressed above on principle and the statutory provisions. An analogy in their context may well be drawn from the Law of Partnership. However, at the very outset I would notice that the position is not identical because partnership envisages a joint or co-ownership of partnership property whereas in a conjugal relationship, as shown above, the spouses may well be the individual and exclusive owners of their respective properties. Nevertheless a marked similarity therein is that in partnership, co-ownership necessarily connotes a jointness of possession of partnership properties whilst the same position inheres in the matrimonial home where the spouses are deemed to be jointly in possession and custody. Now, barring some ancient notes of discordance, it seems to be now well accepted that a partner cannot be held guilty of criminal breach of trust qua partnership property except by virtue of a special agreement either written or conclusively established. This had always been so in English law until it was specifically and altered by Statute 31 and 32 Victoria c. 116 and it is now governed by the special provisions of the same and subsequent legislation. In India, however, in the absence of any statutory change, the legal position would continue to be the same. This came up for pointed consideration before a Full Bench of five Judges in Bhuban Mohan Das v. Surendra Mohan Das, AIR 1951 Cal, 69. The relief sought therein of quashing the proceedings under S. 406, Indian Penal Code, betwixt partners, was granted whilst holding that a charge under S. 406, Indian Penal Code cannot be framed against a person who, according to the complainant, is a partner with him and is accused of the offence in respect of property belonging to them as partners. P.B. Mukharji, J. in his concurring judgment observed as under (Para 46) : "The question here is of much broader application and of a more fundamental nature. Its fundamen- 238

tal nature is this that the very conception of partner ship precludes possibility of entrustment or dominion of the partnership property by one partner as against the other and, therefore, precludes any possible operation of the crime under Section 406 Penal Code, of criminal breach of trust by one partner against the other in respect of the partnership property." The aforesaid view has been expressly referred to and approved by their Lordships in Velji Raghavji v. State of Maharashtra,(1) with the following added observations (at pp. 1435-36) :-

"... Every partner has dominion over property by reason of the fact that he is a partner. This is a kind of dominion which every owner of property has over his property. out it is not dominion of this kind which satisfies the requirements of S.405. In order to establish `entrustment of dominion' over property to an accused person the mere existence of that person's dominion over property is not enough. It must be further shown that his dominion was the result of entrustment. Therefore, as rightly pointed out by Harris, C.J., the prosecution must establish that dominion over the assets or a particular asset of the partnership was, by a special agreement between the parties entrusted to the accused person. If in the absence of such a special agreement partner receives money belonging to the partnership he cannot be said to have received it in a fiduciary capacity or in other words cannot be held to have been 'entrusted' with dominion over partnership properties. "

If that is so in the partnership relation it appears to me that it would be more so in the conjugal relationship with regard to the property within the matrimonial home." "46..... The nature, character and the incident of property within the matrimonial home, so long as the marriage subsists, seem to be such that except by a special written agreement, no entrustment or dominion etc. Of the individual property of the spouses to each other can b e presumed. Equally, herein the specific and ascertainable

(I) A.T.R. 1965 S.C. 1433

239

property of each spouse within the matrimonial home can , be so equivocal and problematic as to oust the requisite mens rea with consequent criminality with regard thereto until the title to such property is clearly and specifically established. If the civil remedy seems to be adequate betwixt partners, during the subsistence of partnership there is no reason why it would not equally be so betwixt spouses in an existing matrimonial home during the subsistence of the conjugal relationship. As already referred to, apart from the civil remedy under the general law, added provisions exist in this context under S.27 of the Hindu Marriage Act buttressed by the procedural provisions of 0.32-A of the Code of Civil Procedure." "47. In view of the above, it would be equally untenable to hold that either the desertion or the expulsion one of the spouses from the matrimonial home would result in entrusting dominion over the property belonging to the other so as to bring the case within the ambit of this pre-requisite under S.405, Indian Penal Code. The joint custody and possession once established would thereafter exclude either express entrustment or the passing of dominion over the property. It was rightly argued that if an irate husband or wife walks out from the matrimonial home in a huff, this cannot constitute an entrustment or dominion over the property to the other. Consequently, unless a special written agreement to the contrary can be established, the strongest presumption arises that during the existence and immediately after the crumbling of the matrimonial home, there was in essence, a joint possession and custody of the property of the spouses therein, including dowry and traditional presents, which would preclude the essentials entrustment of dominion over the property which form the corner-stone of criminality under S.405, Indian Penal Code."

"49. Equally the common use and enjoyment of certain articles of dowry and traditional presents, by the other members of a joint family with the leave and licence of a Hindu wife, cannot have the effect of extending the jointness of control and custody of the couple to undefined

240

and unreasonable limits. Consequently, there is no reason to assume that the mere use or enjoyment of dowry by other members of the household, would have the effect of passing the possession and control thereof Jointly to the Hindu Undivided Family a such." "50. In the aforesaid context, pointed reference must be made to the opening word 'whoever' of S.405 of the Code to highlight that the criminal law does not take ken of any proximity of relationship for the offence of breach of trust. "Whoever" would include within its ambit the parents-in-law, the brothers-in-law, sisters- in-law (and other close relations of the husband) of a Hindu wife provided that the basic ingredients of entrustment or passing of dominion over her separate individual property stands fully satisfied. Apart from the peculiarity of the conjugal relationship and the consequent sharing of the matrimonial home, the existence of the blood relationship of the parties does not seem to be relevant for the applicability or otherwise of S.406 of the Code, Since the other members of the Hindu Joint family, to which the husband may belong, would not be covered by the presumption of jointness of custody v and possession of their individual properties by the spouses alone, they cannot by the mere fact of kinship be excluded from the scope of ss. 405 and 406 of the Code."

"56. To conclude, it necessarily follows from the aforesaid discussion that the very concept of the matrimonial home cannotes a jointness of possession and custody by the spouses even with regard to the movable properties exclusively owned by each of them. It is, therefore, inapt to view the same in view of the conjugal relationship as involving any entrustment or passing of dominion over property day-to-day by the husband to the wife or vice versa. Consequently, barring a special written agreement to the contrary, no question of any entrustment or dominion over property would normally arise during coverture or its imminent break-up. There fore, the very essential pre-requisites and the core ingredients of the offence under S.406 of the Penal Code would be lacking in a charge of criminal breach of trust of

241

property by one spouse against the other. Inevitably, therefore, the purported allegations of breach of trust betwixt husband and wife so long as the conjugal relation ship lasts and the matrimonial home subsists, cannot constitute an offence under Section 406 of the Indian Penal Code, subject to any special written agreement. Equally, as against the close relations of the husband, no facile presumption of entrustment and dominion over the dowry can be raised prims facie and this inevitably has to be by a subsequent conscious act of volition which must be specifically alleged and conclusively established by proof. Lastly, because of the definition in S. 2 of the Dowry Prohibition Act, the offences under the said Act cannot come within the ambit of S. 406 of the Indian Penal Code as these cannot stand together on the same set of facts." "57. Hence the answer (to the question) posed at the very outset is rendered in the affirmative. The bond of matrimony, therefore, bar the spectre of the criminal breach of trust qua the property of the spouses at the very threshold of the matrimonial home. It cannot enter its hallowed precincts except through the back door of a special written contract to the contrary with regard to such property."

I have extracted above several passages from the Judgment of the learned judges of the Full Bench in Vinod Kumar's case (supra) since I share their view and concern for peace and harmony in matrimonial homes and feel that the learned Single Judge who has quashed the wife's complaint in the present case was justified in relying heavily upon that judgment of the Full Bench. In these circumstances, 1 think that in the absence of a separate agreement and specific entrustment by the wife to the husband and of his relations and vice versa of the property of the husband to the wife and or her relation, it would not be possible to draw an inference of entrustment of custody or dominion over the property of one spouse to the other and his or her relations so as to attract the stringent provisions of s.406 I.P.C. The offense of criminal breach of trust is cognizable and non-bailable and punishable with imprisonment for a term of three years or

242

with fine or with both. In the absence of such a separate agreement for specific entrustment of the property of either spouse the appropriate remedy would appear to be by way of a civil suit where there is scope for the parties to the marriage coming together at the instance of relations, elders and well-wishers and patching up their differences. Entertaining complaints of the irate wife or husband against the husband or wife without even an allegation of a specific and separate agreement constituting entrustment of the property of the wife or the husband would have disastrous effects and consequences on the peace and harmony which ought to prevail in matrimonial homes. It is seen from para 45 of-the judgment in Vinod Kumar's case (supra) that in spite of diligent research no instance of any case of successful prosecution of the husband of wife at the instance of the wife or the husband could be brought to the notice of the learned Judges. It may be stated that none was brought to the notice of this Court either in the course of the arguments in this appeal. This would show that the spouses had not lightly rushed in the past to criminal courts with complaints of criminal breach of trust against the other spouses though in the day-to-day life there must have been numerous instances where the wife had used the property or cash of the husband for purposes different from the one for which they were given by the husband to be applied by the wife and vice-versa. I am anxious that no light-hearted change should be brought about in-the position and that the minimum requirement in such cases is a specific separate agreement whereby the property of the wife to husband was entrusted to the husband or wife and or his or her close relations. In the absence of such a specific separate agreement in the complaint, in the present case, I am of the opinion that the learned Single Judge was perfectly justified in following the decision of the Full Bench in Vinod Kumar s case (supra) and quashing the wife's complaint filed against the husband and his close relations. I would, therefore, dismiss the appeal.

In view of the majority decision, this appeal is allowed, the judgment of the High Court is set aside and the complaint filed by the appellant is restored. The accused may now be summoned and put on trial in accordance with law. S.R. Appeal allowed

 

243Supreme Court of India

Pratibha Rani vs Suraj Kumar & Anr on 12 March, 1985
Equivalent citations: 1985 AIR 628, 1985 SCR (3) 191
Bench: Fazalali, S Murtaza

PETITIONER:

PRATIBHA RANI

Vs.

RESPONDENT:

SURAJ KUMAR & ANR.

DATE OF JUDGMENT12/03/1985

BENCH:

FAZALALI, SYED MURTAZA

BENCH:

FAZALALI, SYED MURTAZA

MUKHARJI, SABYASACHI (J)

VARADARAJAN, A. (J)

CITATION:

1985 AIR 628 1985 SCR (3) 191

1985 SCC (2) 370 1985 SCALE (1)458

CITATOR INFO :

RF 1986 SC 833 (50)

RF 1992 SC 604 (103)

ACT:

Nature, character and concomitants of stridhan-Right of exclusive ownership over the stridhan during coverture- Whether the dowry/stridhan given to a wife and her exclusive property becomes a joint property/partnership property by a fiction of being placed in the custody of her husband and her relations, the moment a married woman enters her matrimonial home-Indian Partnership Act, 1932, section 4. Dowry Prohibition Act (28 of 1961) sections 2 Hindu Marriage Act, 1955 section 27 Hindu Succession Act Section 14 Indian Penal Code, sections 405, 406 and 482.

Remedies open under law-Whether criminal remedy is barred when civil remedy is available simultaneously. Entrustment-Charge Or Criminal breach of trust by wife against her husband and his close relations maintainability- Essential ingredients of an offence section 405/406 Indian Penal Code.

Inherent powers of the High Court to quash a First Information Report on a complaint under section 482' the Code of Criminal Procedure, 1973 (Act 11 of 1974), explained.

HEADNOTE:

The appellant Pratibha Rani, the estranged wife of the first respondent Suraj Kumar, filed a criminal complaint against her husband, his father, his three brothers and a brother-in-law in the court of the Additional Chief Judicial Magistrate, Ludhiana, alleging; (i) that she was married to the first respondent at Ludhiana on 4 2. 1972 according to Hindu rites and customs; (ii) that the aforesaid persons, namely, father, brothers and brother-in-law of the first respondent attended the marriage and demanded dowry from the appellants' parents as consideration for the marriage; (iii) that the dowry articles mentioned in the list worth Rs 60,000 in the form of gold ornaments, clothes and other valuables were given and entrusted to the respondents and four others at Ludhiana at the time of 'doli' on 5. 2. 1972 in the presence of Kapur Chand Jain and six others; (iv) that all the six respondents, from the time of marriage started teasing, harassing and beating her and they kept her without even food to extract more money from her parents; (v) that they turned out the appellant with her children in the beginnings of 1977 (vi) that after a great deal of persuasion and intervention by Panchayatdars, respondent No. I came

192

to Ludhiana and took her to his house, after giving an undertaking in writing on 21. 6. 1977 not to misbehave with and not to maltreat the appellant and her children; (vii) that after some time all the respondents in the Complaint not only started again maltreating the appellant and misbehaving with her, but also brought the appellant at 4.30 a.m. On 11.12.80 and left her near Kailash Cinema Chowk, (viii) that the articles (the stridhana) mentioned in the list appended to the complaint were never given by the respondents to the appellant for her use but were retained by them illegally and with the dishonest intention of causing wrongful gain to themselves and wrongful loss to the appellant y (ix) that when the appellants' husband and his brother, Vishwinder Kumar, respondent 1 and 5 in the complaint, came to Ludhiana on 10 2.81 to attend the proceedings started by the appellant under section 125 Criminal Penal Code her parents persuaded them to return the articles entrusted to them at the time of the marriage but they flatly refused to comply with that demand; (x) that the articles have not been returned in spite of service of notice dated 17.12.81 on the first respondent; (xi) that the respondents in the complaint have dishonestly, thus, converted the articles belonging to the appellant for their use in violation of the instructions of the appellants' parents given at the time of the marriage to give the articles for the appellants' use and that (xii) they individually and jointly committed the offences under sections 405 and 406 Indian Penal Code.

Thereupon respondent No. 1 filed Criminal Misc. Application No. 4876 of 1981 in the Punjab and Haryana High Court under section 482 of the Code of Criminal Procedure for quashing the criminal proceedings and the complaint taken on file by the Additional Chief Judicial Magistrate, Ludhiana under section 406 IPC and his order summoning them.A Learned Single Judge of the High Court relying strongly upon the observations made by a Full Bench of that High Court in Vinod Kumar Sethi & Ors. v. State of Punjab & Ors. reported in AIR 1982 Punjab 372 allowed the petition and quashed the proceedings arising out of the appellants' complaint, observing that the allegations in the appellants' complaint are similar to the one in that case and therefore, fully covered by the ratio in that decision. Hence the appeal by special leave.

Allowing the appeal, the Court,

^

HELD; (Per E.lzal Ali, J.) (on behalf of Sabyasachi Mukharji, J. and himself)

1.1 The stridhan property of a married woman cannot acquire the character of a joint property of both the spouses as soon as she enters her matrimonial home so as to eliminate the application of section 406 IPC. The position of stridhan of a Hindu married woman's property during coverture is absolutely clear and unambiguous; she is the absolute owner of such property and can deal with it in any manner she likes-She may spend the whole of it or give it away at her own pleasure by gift or will without any reference to her husband. The entrustment to the husband of the stridhan property is just like something which the wife keeps in a bank and can withdraw any amount when ever she likes without any hitch or hindrance. Ordinarily, the husband has no right or interest in it with the sole exception that in times of extreme distress, as 193

in famine, illness or the like, the husband can utilize it but he is morally bound to restore it or its value when he is able to do so. This right is purely personal to the husband and the property so received by him in marriage cannot be proceeded against even in execution of a decree for debt. [206F; 201D-E]

Suraj Kumar & Anr. v. Pratibha Rani, Criminal Misc. Petition No. 4876 of 1981 Punjab & Haryana High Court reversed.

Vinod Kumar Sethi & Ors. v. State of Punjab & Anr. AIR 1982 Punjab 372; Surinder Mohan v. Smt Kiran Saini, 1977 Chandigarh Law Reporter 212; Kailash Vati v. Ayodhya Parkash, ILR (1977) 1 Punjab & Haryana 642 (FB) overruled. 1.2 A perusal of the allegations made in the complaint undoubtedly makes out a positive case of the accused having dishonestly misappropriated the articles handed over to the n in a fiduciary capacity. To characterise such an entrustment as a joint custody or property given to the husband and the parents is wholly unintelligible.A perusal of the list reveals that so far as the jewellery and clothes, blouses, nighties and gowns are concerned they could be used only by the wife and were her stridhan. By no stretch of imagination could it be said that the [ornaments and sarees and other articles mentioned above could also be used by the husband. If, therefore, despite demands these articles were refused to be returned to the wife by the husband and his parents, it amounted to an offence of criminal breach of trust. All the ingredients of an offence under section 405 IPC were pleaded and a prima facie case for summoning the accused was made out. In such circumstances, the complaint should have been given an opportunity by the High Court to prove her case rather than quashing the complaint. Such an exercise of jurisdiction by the High Court under section 482 Cr. P. is totally unwarranted by law. [203A; 204B-D; 203B-C] 2.1 Criminal law and matrimonial home are not strangers. Crimes committed in matrimonial home are as much punishable as anywhere else. The mere factum of the husband and wife living together does not entitle either of them to commit a breach of criminal law and if one does then he/she will be liable for all the consequences of such breach. In the case of stridhan properly also, the title of which always remains with the wife though possession of the same may sometimes be with the husband or other members of his family, if the husband or any other member of his family commits such an offence, they will be liable to punishment for the offence of criminal breach of trust under sections 405 and 406 IPC. Just as a newly married woman living in the same house and under the same roof cannot be expected to keep her personal property or belongings like jewellery, clothing, etc. under her own lock and key thus showing a spirit of distrust to the husband at the very behest, a husband cannot be permitted to cast his covetous eyes on the absolute and personal property of his wife merely because it is kept in his custody, thereby reducing the custody to a legal farce. On the other hand, even if the personal property of the wife is jointly kept it would be deemed to be expressly or impleedly kept in the custody of the husband and i f he dishonestly misappropriates or refuses 194

to return the same, he is certainly guilty of criminal breach of trust, and there can be no escape from this legal consequence. [207E-G; 208B-Cl

2.2 It is an anathema to suppose that when a civil remedy is available, a criminal prosecution is completely barred. The two remedies-are under civil law and the other under criminal law are not mutually exclusive but coextensive and essentially differ in their content and consequences. Therefore, it cannot be said that, if the husband dishonestly misappropriates the stridhan property of his wife though kept in his custody, that would not par prosecution under section 406 IPC or render the ingredients of section 405 IPC nugatory or abortive. To say that because the stridhan of a married woman is kept in the custody of her husband no action against him can be taken as no offence is committed is to override and distort the real intent of law. [208E-F]

3.1 Neither section 27 of the Hindu Marriage Act nor section 14 of the Hindu Succession Act, go to the extent of providing that the claim of a woman on the basis of stridhan is completely abolished, or that a remedy under the criminal law for breach of trust is taken away. All that the two sections, provide is that if the husband refuses to return the stridhan property of his wife, it will be open to the wife to recover the same by a properly constituted suit. [204G-H; 205A]

3.2 Section 27 of the Hindu Marriage Act merely provides for- an alternate remedy and does not touch or affect in any way the criminal liability of the husband in case it is proved that he has dishonestly misappropriated the stridhan of his wife. It cannot also be spelt out from any textbook or the sastric law of the Hindus that these two Acts take away the stridhan right of a woman-at the most these Acts merely modify the concept of stridhan. [205C-D]

Bhai Sher Jang Singh & Anr. v. Smt. Virinder Kaur, 1979 Criminal Law Journal 493 approved.

Surinder Mohan v. Smt. Kiran Saini, 1977 Chandigarh Law Reporter 212 over ruled.

4.1 It is neither appropriate nor apposite to import the concept of partner ship of husband and wife for the simple reason that the concept of partnership is entirely different from that of the husbands' keeping the stridhan in his custody. From the definition of the partnership in section 4 of the Indian Partnership Act, it is manifest that in a partnership the wife must by some clear and specific act indicate that the stridhan which has been entrusted to the husband is to be used for a partnership business and the losses of the firm, if any, would have to be shared by both.A pure and simple act of entrustment of the stridhan to the husband does not attract any of the essential ingredients of a partnership as defined in the Partnership Act. When the essential conditions of a partnership do not exist. the mere factum of entrustment of stridhan would not constitute any co-ownership or legal partnership, There is also no

195

question of the wife, constituting herself a partner with her husband merely by allowing him to keep the article or money in his custody. Further, in this case, there is, neither any pleading nor any allegation that after her marriage, the appellant transferred all her properties to her husband for carrying on a partnership business in accordance with the provisions of the Partnership Act. Therefore, a criminal prosecution under section 406 IPC is maintainable.

[209E; 210B-C; G; 211C-D]

Vinod Kumar Sethi & Ors. v. State of Punjab & Anr. AIR 1982 Punjab 372; Surinder Mohan etc. V. Smt. Kiran Saini, 1977 Chandigarh Law Reporter 212; Kailash Vati v. Ayodhya Parkash, ILR (1973) 1 Punjab & Haryana, P 612; Kailash Nath Agarwal & Ors. v. Prem Pal Agarwal & Anr. Crl. Misc. case No. 676 of 1981 connected with Crl. Misc. case No. 2753 of 1981 decided on 22.12.83 Allahabad High Court overruled. In the instant case, however, there is neither any allegation nor anything in the complaint to show that when the wife entered her matrimonial home she had entrusted property to her husband so as to make him part owner of the same. Therefore, the question Or the husband having dominion over the property does not at all arise. In fact the wife has nothing to do With the partnership, if any and the husband is a pure and simple custodian of the property and cannot use the Same for any purposed without her consent. [210E-F]

The concept of stridhan property of a married woman becoming joint property of both the spouses as soon as she enters her matrimonial home and continues to be so until she remains there or even if there is a break in the matrimonial alliance, is in direct contravention of Hindu law of Sadayika which has been administered since more than a century by High Court, Privy Council and also the Supreme Court. [212C-D]

4.2 The Full Bench decision in Vinod Kumar's case would not only render the provisions of section 406 IPC inapplicable and nugatory even if the husband has the audacity or the importunity of refusing to return the stridhan of his wife, but also be in direct contravention of a long course of decisions of Supreme Court on the ingredients of section 405 IPC. [212A-B]

By a pure and simple figment of the fertile imaginations, the Judges in the Vinod Kamat's case seem to have rewritten the law of criminal breach of trust contained in sections 405 and 406 IPC so as to carve out an imaginary exception to the application of the Penal Code- more tragic consequence of the view taken by the High Court is that even if there is a break in the matrimonial alliance and the wife wants her husband to return her exclusive property and he refuses lo return even then the provisions of section 406 IPC would not apply. It is an extreme travesty of justice for a court to say that whenever a married demands her stridhan property from her husband she should be driven to the dilatory process of a civil court and her husband would be debarred from being prosecuted by a criminal court. By a strange and ingenious process of holding that such an act of a husband does not attract the provisions of the 196

Penal Code, as the property being joint there is no question of the husband being a trustee or holding the same in a fiduciary capacity. Such a view is not only contradictory but-what the High Court has said before regarding the applicability of section 27 of the Hindu Marriage Act and the nature of stridhan-is also neither in consonance with logic and reason nor with the express provisions of the Penal Code and seems to be inspired by a spirit of male chauvinism so as to exclude the husband from criminal liability merely because his wife has refused to live in her matrimonial home. The High Court, functioning in a civilised and socialistic society such as ours cannot play such a havoc with judicial interpretation of an important branch of law. The High Court cannot make a complete volte-face by holding that these very properties after marriage become joint property of both the spouses. The High Court has not realised that the theory or philosophy of matrimonial home propounded by it stands directly contradicted by its own observations.

[212D-H; 213A; H; 214A]

4.3 The fundamental core of the offence of criminal breach of trust is that a property must be entrusted and the dominion of the property should be given to the trustee. In the present case, all these conditions, even according to the findings of the High Court though not its conclusions are clearly established.

[217C]

Chelloor Manaklal Narayan Ittiravi Nambudiri v. State of Travancore; AIR 1953 SC478; Jaswantrai Manilal Akhaney v. State of Bombay, [1956] SCR 483; State of Gujarat v. Jaswantlal Nathalal[1968] 2 SCR 408; Sushil Kumar Gupta v. Joy Shankar Bhattacharjee, AIR 1971 SC 1543; Superintendent JUDGMENT:

[4] SCC 230 referred to.

Harihar Prasad Dubey v- Tulsi Das Mundhra & Ors. AIR 1949 Calcutta 207; Akharbhai Nasarali v. Md. Hussain Bhai AIR 1961 MP 37; Basudeb Patra v. Kana. Lal Haldar, AIR 1949 Calcutta 207, Bhai Sher Jang Singh and Anr. v. Smt. Virinder Kaur, 1979 Crl. L-J. 493; Avtar Singh and Anr v. Kirpal Kaur, Crl. Misc. No. 2144 of 1979 and Cr l Misc. No. 2145 of 1979 approved.

Vinod Kumar Sethi & Ors. v- State of Punjab and Anr. ATR 1982 Punjab 372; Surinder Mohan etc. v. Smt. Kiran Saini, 1977 Chandigarh Law Reporter 212; Kailash Nath Agarwal & Ors- v. Prem Pal Agarwal & Anr. Crl. Misc. Case No. 676 of 1981 connected with Crl. Misc. case No. 2753 of 1981, Allahabad High Court: Kailash Vati v. Ayodhya Parkash, ILR (1977) 1 Punjab d: Haryana 642 overruled.

5. For the purpose of exercising its power under section 482 Cr. PC to quash a First Information Report or a complaint the High Court would have to proceed entirely on the basis of the allegations made in the complaint or the documents accompanying the same per se. It has no jurisdiction to examine the correctness or otherwise of the allegations. In case no offence is committed on the allegation and the ingredients of section 405 and 406 IPC are not made out, the High Court would be justified in quashing the proceedings. In the -present case, the allegations are both clear, specific and unambiguous and 197

therefore, the complaint should have been given a chance to prove her case. It is, of course open to the accused at the trial to take whatever defences that were open to him or her but that stage had not yet come and therefore, the High Court was totally ill-advised to speculate on the merits of the case at that stage and quash the proceedings. Since all the facts stated in the complaint constituted an offence under section 406 IPC, the appellant cannot be denied the right to prove her case at the trial by pre-empting it the very behest by the order passed by the High Court. [223D-H; 224D-E-]

Vinod Kumar Sethi & Ors. v. State of Punjab & Anr, AIR 1982 Punjab 372, over-led.

L.V. Jadhav v. Shakarrao Abasaheb Pawar & Ors. AIR 1983 SC 1219; Smt. Nagawa v. Veeranna Shivalingappa Konjalgi & ors. [1976] Supp. SCR 123 applied.

OBSERVATION

(It is surprising to find that so deeply drowned and inherently engrossed are some of the High Courts in the concept of matrimonial home qua the stridhan property- of a married woman that they simply refuse to believe that such properties are meant for the exclusive use of the wife and could also be legally entrusted to the husband or his relations. Thus, if the husband or his relations misappropriate the same and refuse to hand it over to the wife and convert them to their own use and even though these facts arc clearly alleged in a complaint for an offence under section 405/406 IPC, some courts take the view that the complaint is not maintainable. Thus even when clear and specific allegations are made in the complaint that such properties were entrusted to the husband, they refuse to believe these hard facts and brush them aside on the ground that they are vague, and completely shut their eyes to the fact that the husband could also be guilty under section 405/406 IPC in view of the clear allegations made in the complaint. In other words, the High Courts simply refuse to believe that there can be any such entrustment and even if it is so, no offence is committed. Such an approach amounts to a serious distortion of the criminal law, resulting in perpetrating grave and substantial miscarriage of justice to the wife at the hands of the High Courts. The Supreme Court cannot continuance such a wrong and perverse approach.) [224G-H, 225A-C]

Per A. Varadarajan, J. (dissenting)

1.1 In the absence of a separate agreement and specific entrustment by the wife to the husband and or his relations and vice versa of the property of the husband to the wife and or her relation, it would not be possible to draw an inference of entrustment of custody or dominion over the property of one spouse to the other and his or her relations so as to attract the stringent provisions of section 406 IPC. The offence of criminal break of trust is cognizable and non-bailable and punishable with imprisonment for a term of three years or with fine or with both. In the absence of such a separate agreement for specific entrustment of the property of either spouse the appropriate remedy would appear to be by way of a civil suit where there is scope for the parties to the marriage coming together at the instance of relations, elders and well- wishers and patching up their differences. [241G-H; 242A] 198

1.2 Entertaining complaints of the irate wife or husband against the husband or wife without even an allegation of a specific and separate agreement constituting entrustment of the property of the wife of the husband would have disastrous effects and consequences on the peace and harmony which ought to prevail in matrimonial homes. [242B] 1.3 The fact that no instance of any case of successful prosecution of the husband or wife at the instance of the wife or the husband could be brought to the notice of the Supreme Court in the course of the arguments in this appeal would show that the spouses had not lightly rushed in the past to criminal courts with complaints of criminal breach of trust against the other spouses though in the day-to-day life. There must have been numerous instance where the wife had used the property or cash of the husband for purposes different from the one for which they were given by the husband to be applied by the wife and vice-versa. Therefore, the minimum requirement in such cases is a specific separate agreement whereby the property of the wife or husband was entrusted to the husband or wife and or his or her close relations. In the absence of such a specific separate agreement in the present case the complaint was rightly quashed. [242D-F]

&

CRIMINAL APPELLATE JURISDICTION: Criminal Appeal No. 684 of 1982

From the judgment and order dt. the 31st May, 1982 of the High Court of Punjab & Haryana at Chandigarh in Crl. Misc. No. 4876M/81.

V.C. Mahajan, and N.S. Das Bahl for the Appellant. Altat Ahamed for the Respondents.

Mrs. U. Kapoor for the Intervener.

T The following Judgments were delivered FAZAL ALI, J. Sometimes the law which is meant to impart justice and fair play to the citizens or people of the count is so torn and twisted by a morbid interpretative process that instead of giving haven to the disappointed and dejected litigants it negatives their well established rights in law. The present case reveals the sad story of a helpless married woman who, having been turned out by her husband without returning her ornaments, money and clothes despite repeated demands, and dishonestly misappropriating the same, seems to have got some relief by the court of the first instance but to her utter dismay and disappointment when she moved the High Court she was forced like a dumb- driven cattle to seek the dilatory remedy of a civil suit- such was the strange and harsh approach of the High Court, with due respect, which seems to have shed all the norms of justice and fair play. Even so, the High Court is not much to be blamed because in the process of following precedents or decisions of doubtful validity of some courts, it tried to follow suit. It may be stated that even the old 199

classic Hindu law jurists and celebrated sages conceded certain substantial rights to the women, one of which was - what is called Saudayika or stridhan, with which we are concerned here.

This now brings us to a brief discussion of the nature, character and concomitants of stridhan. In the instant case, we are mainly concerned with that part of stridhan which is the absolute property of a married women during coverture. Sir Gooroodas Banerjee in 'Hindu Law of Marriage and Stridhana' while describing the nature of stridhan quoted Katyayana thus:

"Neither the husband, nor the son, nor the father, nor the brother, has power to use or to alien the legal property of a woman. And if any of them shall consume such property against her own consent he shall be compelled to pay its value with interest to her, and shall also pay a fine to the king... Whatever she has put amicably into the hands of her husband afflicted by disease, suffering from disease, or sorely pressed by creditors, he should repay that by his own freewill. " (P.341)

At another place while referring to the nature of a husband's rights over stridhan during coverture, the author referring to Manu says thus:

" .. and by the law as expounded by the commentators of the different schools, the unqualified dominion of the husband is limited to only some descripttions of the wife's property, while as regards the rest he is allowed only a qualified right of use under certain circumstances specifically defined." (p.340)

Similarly, while describing the nature of stridhan generally, which is known as saudayika, the author says thus:

"First, take the case of property obtained by gift. Gifts of affectionate kinderd, which are known by the name saudayika stridhana, constitute a woman's absolute property, which she has at all times independent power to alienate, and over which her husband has only a qualified right, -namely, the right of use in times of distress."

200

The entire classical text on the subject has been summarised by N.R. Raghavachariar in 'Hindu Law' (5th Edn) at page 533 (section 487) where the following statement is made:

"487. Powers During Coverture.

Saudayika, meaning the gift of affectionate kindred, includes both Yautaka or gifts received at the time of marriage as well as its negative Ayautaka. In respect of such property, whether given by gift or will, she is the absolute owner and can deal with it in any way she likes. She may spend, sell or give it away at her own pleasure by gift or will without reference to her husband and property acquired by it is equally subject to such rights. Ordinarily, the husband has no manner of right or interest in it. But in times of extreme distress, as in famine, illness or imprisonment, or for the performance of indispensable duty the husband can take and utilise it for his personal purposes, though even then he is morally bound to restore it or its value when able to do so. But this right is purely personal to him and cannot be availed of by a holder of a decree against the husband, and if the husband dies with out utilising the property for the liquidation of his debts, his creditors cannot claim to proceed against it in the place of her husband."

To the same effect is Maines' treatise on Hindu Law at page 728. The characteristics of Saudayika have also been spelt out by Mulla's Hindu law at page 168 (section 113) which gives a complete list of the stridhan property of a woman both before and during coverture, which may be extracted thus:

"113. Manu enumerates six kinds of stridhana:

1. Gifts made before the nuptial fire, explained by Katyayana to mean gifts made at the time of marriage before the fire which is the witness of the nuptial (adhyagni).

2. Gifts made at the bridal procession, that is, says Katyayana, while the bride is being led from the residence of her parents to that of her husband 201

(adhyavanhanika)

3. Gifts made in token of love, that is, says Katyayana, those made through affection by her father-in-law and mother-in-law (pritidatta), and those made at time the of her making obeisance at the feet of elders (padavan danika).

4. Gifts made by father.

5. Gifts made by mother.

6. Gifts made by a brother."

It is, therefore, manifest that the position of stridhan of a Hindu married woman's property during coverture is absolutely clear and unambiguous; she is the absolute owner of such property and can deal with it in any manner she likes - she may spend the whole of it or give it away at her own pleasure by gift or will without any reference to her husband. Ordinarily, the husband has no right or interest in it with the sole exception that in times of extreme distress, as in famine illness or the like, the husband can utilise it but he is morally bound to restore it or its value when he is able to do so. It may be further noted that this right is purely personal to the husband and the property so received by him in marriage cannot be proceeded against even in execution of a decree for debt.

Such being the nature and character of stridhan of a woman, it is difficult to countenance the view of the Punjab & Haryana High Court in Vinod Kumar Sethi & Ors. v. State of Punjab & An.(l) that the stridhan property of a married woman becomes a joint property as soon as she enters her matrimonial home. We shall deal with this aspect of the matter a little later.

We would first like to narrate the facts of the case to show how the complaint filed by the appellant was wrongly quashed by the High Court. The general allegations made in the complaint may be summarised as follows:- (1) AIR 1982 Punjab 372-

202

The complainant was married to Suraj Kumar, Accused No. 1 (respondent) on 4.2.72 at Ludhiana according to Hindu rites and customs in the presence of respectable persons. Accused No.2 was the father and accused Nos.3 to 5 were brothers and No.6 was brother-in-law of accused No.; It is further alleged that all the accused attended and actively participated in the marriage of the complainant and demanded dowry. The must important allegation made by the appellant was that her parents and relatives gave by way of dowry articles worth Rs. 60,000/- inclusive of gold ornaments, clothes and other things which were entrusted to accused Nos.1 to 6 on 5.2.72 which were taken into possession by them. Soon after the marriage, accused No. 1 started harassing, teasing and beating the complainant and ultimately turned her out alongwith her children sometime in the year 1977. It was avered in para 4 of the complaint that accused never returned the articles to her, the relevant portion of the allegations may be extracted thus:- "The articles above-mentioned were never given by the accused to the complainant for her use and possession of the same was illegally, dishonestly and malafidely retained by the accused in order to make a wrongful gain to them selves and wrongful loss to the complainant.

The accused refused to give the entrusted articles of dowry, which were the stridhan of the complainant. On 10.2.1981 when the accused Nos. 1 to 5 came to Ludhiana to attend the proceeding u/s 125 Cr.P.C., filed by the complainant in the Court of Shri S.S. Tiwana, they were persuaded by the parents of the complainant to send the articles entrusted to them at the time of marriage but they gave flat refusal to its notice which was served upon the accused No.1 which was dated 17.12.80, but to no effect. The accused have thus dishonestly used and converted the articles aforementioned to their own use, who are still in possession of the same in violation of the direction given by the parents of complainant. The parents of the complainant directed the accused at the time of marriage to give the articles to the complainant for her use, in the presence of the aforesaid persons, but the accused have not done the needful of the demand and have thus committed criminal breach of trust punishable u/s 406 IPC."

203

A perusal of the allegations made in the complaint undoubtedly makes out a positive case of the accused having dishonestly misappropriated the articles handed over to them in a fiduciary capacity. To characterise such an entrustment as a joint custody or property given to the husband and the parents is wholly unintelligible to us. All the ingredients of an offense under s.405 IPC were pleaded and a prima facie case for summoning the accused was made out. In such circumstances, the complainant should have been given an opportunity by the High Court to prove her case rather than quashing the complaint. Such an exercise of jurisdiction under s.482 Cr.P.C. is totally unwarranted by law. We might also mention that alongwith the complaint, a list of valuable articles had also been given, the relevant portion of which may be extracted thus;

I. " Jewellery"

1. Nine complete gold sets

2. One complete diamond set

3. Three gold rings

4. Two golden Bahi (Baju Band)

5. One golden chain

6. One shingar patti with golden tikka

7. One golden nath (Nose ring)

8. Twelve golden bangles

II. Silver articles

1. Six glasses and one jug

2. Two surma danies

3. One tagari

4. Two payals

III. Clothes

Fifty one sarees, twenty one suits alongwith petti 204

coats, blouses, nighties, shawls, sweaters, night suits, gowns and woollen coat etc., six complete beds with sheets, etc."

A perusal of the list reveals that so far as the jewellery and clothes, blouses, nighties and gowns are concerned they could be used only by the wife and were her stridhan. By no stretch of imagination could it be said that the ornaments and sarees and other articles mentioned above could also be used by the husband, If, therefore, despite demands these articles were refused to be returned to the wife by the husband and his parents, it amounted to an offence of criminal breach of trust. In mentioning the articles in the list, we have omitted furniture and utensils which though also belonged to the complainant yet there is some room for saying that these were meant for joint use of the husband and wife.

Thus, the facts mentioned in the complaint taken at their face value reveal a clear allegation that the stridhan property of the appellant was entrusted to the husband who refused to return the same to her

Some courts were of the opinion that in view of s. 27 of the Hindu Marriage Act and s. 14 of the Hindu Succession act, the concept of stridhan property of a woman was completely abolished. For instance, the Punjab & Haryana High Court in a case reported in Surindra Mohan etc. v. Smt. Kiran Saini(1) held thus:

"That under the present law on claim can be made on the basis of stridhan, as it has now been completely abolished and cannot avail against statute which makes it the joint property of the parties."

We are of the opinion that this view of the High Court is not legally sustainable because neither of the two Acts, referred to above, go to the extent of providing that the claim of a woman on the basis of stridhan is completely abolished. All that the two sections, mentioned above, provide is that if the husband re-

(1) 1977 Chandigarh Law Report 212

205

fuses to return the stridhan property of his wife, it will be open to the wife to recover the same by properly constituted suit. The sections nowhere provide that the concept of stridhan is abolished or that a remedy under the criminal law for breach of trust is taken away. In a later decision in Bhai Sher Singh & Anr. v. Smt. Virinder Kaur(1), it was very rightly pointed out by the same High Court that s. 27 of the Marriage Act merely provides an alternate remedy to the wife to bring a properly constituted suit in respect of the stridhan property which the husband refused to return. Thus, it is clear that s. 27 merely provides for an alternate remedy and does not touch or affect in any way the Criminal liability of the husband in case it is proved that he has dishonestly misappropriated that stridhan of his wife. It cannot also be spelt out from any textbook or the sastric law of the Hindus that the two Acts mentioned above take away the stridhan right of a woman-at the most these Acts merely modify the concept of stridhan. It may be useful to refer to certain pertinent observations in the aforesaid case.

"The aforementioned passage shows that a female has an absolute right to use her stridhan in any way she likes and even if her husband can take this property at the time of distress, this right is personal to him, The allegations made in the instant complaint are not that the husband of the respondent has placed her ornaments and jewellery etc. Out of her way. What has been alleged therein is that the petitioners who are the parents-in-law of the respondent have converted the ornaments and clothes, etc. presented to the respondent at the time of her marriage to their own use.

Section 27 of the Hindu Marriage Act empowers a Court while deciding a matrimonial dispute to also pass a decree in respect of property which may jointly belong to both the husband and the wife. This section at best provides a civil remedy to an aggrieved wife and does not in any way take away her right to file a crimi-

(1) 1979 Crl. L.J. 493.

206

nal complaint if the property belonging to her is criminally misappropriated by her husband.'' In these circumstances, the decision reported in 1977 Chandigrah Law Reporter 212 can no longer be considered good law. Even in Vinod Kumar's case (supra) the Full Bench reiterated the view that s. 27 in no way abolishes stridhan but expressly recognises the property exclusively owned by the wife. In this connection, the Court observed thus: "The express words of the provision refer to property 'which may belong jointly to both the husband and the wife'. It nowhere says that all the wife's property be longs jointly to the couple or that Stridhan is abolished and she cannot be the exclusive owner thereof. Indeed, in using the above terminology the statute expressly recognises that property which is exclusively owned by the wife is not within the ambit of Section 27 of the Hindu Marriage Act- - -Equally no other provision in the Hindu Marriage Act could be pointed out which erodes the concept of Stridhan or in anyway incapacitates the Hindu wife to hold property as an exclusive owner."

The sheet-anchor of the arguments of the counsel for the respondents-which is based on the decision of the Punjab & Haryana High Court in Vinod Kumar's case-is that the moment a woman after Marriage enters her matrimonial home, her stridhan property becomes a joint properly of both the spouses and the question of application of s. 406 I.P.C is completely eliminated. It is true that to a great extent this part of the argument of the learned counsel is supported by the aforesaid decision but, in our opinion, the decision, so far as this aspect of the matter is concerned, is wholly unsustainable. We would first extract the exact ratio held by the High Court in Vinod Kumar's case: "To conclude, it necessarily follows from the aforesaid discussion that the very concept of the matrimonial home connotes a jointness of possession and custody by the spouses even with regard to the moveable properties exclusively owned by each of them. It is, therefore, inapt to view the same in view of the conjugal relationship as

207

involving any entrustment or passing of dominion over property day-to-day by the husband to the wife or vice versa. Consequently, barring a special written agreement to the contrary, no question of any entrustment or dominion over property would normally arise during coverture or its imminent break-up. Therefore, the very essential prerequisites and the core ingredients of the offence under S.406 of the Penal Code would be lacking in a charge of criminal breach of trust of property by one spouse against the other."

These observations on doubt support the contention of the learned counsel for the respondent but we find it impossible to agree with the aforesaid observations for the reasons that we shall give hereafter. We fail to understand the logic of the reasoning adopted by the High Court in investing the pure and simple stridhan of the wife with the character of a joint property. We are surprised that the High Court should have taken the view that a woman's absolute property though well recognised by law is interpreted by it as being shorn its qualities and attributes once a bride enters her matrimonial home. We are clearly of the opinion that the mere factum of the husband and wife living together does not entitle either of then to commit a breach of criminal law and if one does then he/she will be liable for all the consequences of such breach. Criminal law and matrimonial home are not strangers. Crimes committed in matrimonial home are as much punishable as anywhere else. In the case of stridhan property also, the title of which always remains with the wife though possession of the same may sometimes be with the husband or other members of his family, if the husband or any other member of his family commits such an offence, they will be liable to punishment for the offence of criminal breach of trust under ss. 405 and 406, IPC.

Afterall how could any reasonable person expect a newly married women living in the same house and under the same roof to keep her personal property or belongings like jewellery, clothing, etc., under her own lock and key, thus showing a spirit of distrust to the husband at the very behest. We are surprised how could the High Court permit the husband to cast his covetous eyes on the

208

absolute and personal property of his wife merely because it is kept in his custody, thereby reducing the custody to a legal farce. On the other hand, it seems to that us even if the personal property of the wife is jointly kept, it would be expressly or impliedly kept in the custody of the husband and if he dishonestly misappropriates or refuses to return the same, he is certainly guilty of criminal breach of trust, and there can be no escape from this legal consequence. The observations of the High Court at other places regarding the inapplicability of s. 406 do not appeal to us and are in fact not in consonance with the spirit and trend of the criminal law. There are a large number of cases where criminal law and civil law can run side by side. the two remedies are not mutually exclusive but clearly coextensive and essentially differ in their content and consequence. The object of the criminal law is to punish an offender who commits an offence against a person, property of the State for which the accused, on proof of the offence, is deprived of his liberty and in some cases even his life. This does not, however, affect the civil remedies at all for suing the wrong deer in cases like arson, accidents, etc. It is an anathema to suppose that when a civil remedy is available, a criminal prosecution is completely barred. The two types of actions are quite different in content, scope and import. It is not at all intelligible to us to take the stand that if the husband dishonestly misappropriates the stridhan property of his wife, though kept in his custody, that would bar prosecution under s. 406 I.P.C. Or render the ingredients of s. 405 IPC nugatory or abortive. To say that because the stridhan of a married woman is kept in the custody of her husband, no action against him can be taken as no offence is committed is to override and distort the real intent of the law.

Coming back to the theory of matrimonial home and the stridhan becoming a joint property of the two spouses, the logical effect of the observation made by the High Court is that once a woman enters her matrimonial home she completely loses her exclusive stridhan by the same being treated as a joint property of the spouses. In other words, if this view is taken in its literal sense the consequence would be to deprive the wife of the absolute character and nature of her stridhan and make the husband a co-owner of the same - such a concept is neither contemplated nor known to Hindu law of stridhan, nor does it appeal to pure

209

common sense. It is impossible to uphold the view that once a married woman enters her matrimonial home her stridhan property undergoes a vital change so as to protect the husband from being prosecuted even if he dishonestly misappropriates the same. For instance, properties like jewellery, clothing, cash, etc. given by her parents as gifts cannot be touched by the husband except in very extreme circumstances, viz., where the husband is in imprisonment or is in serious distress. Even then the religion and the law enjoins that the husband must compensate the wife and if he cannot do so, he must pay fine to the King which means that the husband would` be liable to penal action under the present law of the land. - One of the arguments addressed by the counsel for the respondent which had appealed to thee full Bench of the Pun jab & Haryana High Court in Vinod Kumar's case (supra) as also to our learned Brother Varadarajan, J., is that after entering the matrimonial home the custody of the stridhan entrusted by the wife to her husband becomes a sort of a partnership firm and in this view of the matter the question of criminal breach of trust does not arise. In our opinion, it is neither appropriate nor apposite to import the concept of partnership in the relationship of husband and wife for the simple reason that the concept of partnership is entirely different from that of the husband's keeping the stridhan in his custody. Section 4 of the Indian Partnership Act, 1932 (hereinafter referred to as the 'Partnership Act') defines 'partnership' thus:

"partnership" is the relation between persons who have agreed to share the profit of a business carried on by all or any of them acting for all.

Persons who have entered into partnership with one another are called individually "partners" and collectively "a firm" and the name under which their business is carried on is called the "firm name". The essential ingredients of a partnership are: (1) that there should be an actual or physical overt act on the part of two persons to embark an a business adventure.

(2) that if any business is carried on by one or any

210

of the partners the profits of the business shall be shared by - them in the ratio contained in the partnership agreement.

It is, therefore, manifest that in a partnership the wife must by some clear and specific act indicates that the stridhan which has been entrusted to the husband is to be used for a partnership business and the losses of the firm, if any would have to be shared by both. In other words, one of the essential conditions of a partnership firm is that every partner must have dominion over the property by virtue of the fact that he is a partner. This aspect of the matter was highlighted in a decision of this Court in Velji a Raghavji v. State of Maharashtra(1) where the following observations were made:

".. Every partner has dominion over property by reason of the fact that he is a partner. This is a kind of dominion which every owner of property has over his property. But it is not dominion of this kind which satisfies the requirements of s. 405. In order to establish 'entrustment of dominion' over property to an accused person the mere existence of that person's dominion over property is not enough. It must be further shown that his dominion . was the result of entrustment."

In the instant case, however, there is neither any allegation nor anything in the complaint to show that when the wife entered her matrimonial home she had entrusted the property to her husband so as to make him part owner of the same. Therefore, the question of the husband's having dominion over the property does not at all arise. In fact, the wife has nothing to do with the partnership, if any, and the husband is a pure and simple custodian of the property and cannot use the same for any purpose without her consent.A pure and simple act of entrustment of the stridhan to the husband does not attract any of the essential ingredients of a a partnership as defined in the Partnership Act.

In the instant case, there is also no question of the wife constituting herself a partner with her husband merely by allowing him to keep the articles or money in his custody. There is neither any pleading nor any allegation that after her marriage, the appe-

(1)AIR 1965 SC 1433.

211

llant transferred all her properties to her husband for carrying on a partnership business in accordance with the provisions of the Partnership Act. Thus, in our opinion, it cannot be said that a bare act of keeping stridhan property in the custody of the husband constitutes a partnership and, therefore, a criminal case under s. 406 IPC is not maintainable. It is not necessary for us to multiply cases on this point on which there does not appear to be any controversy. We have already pointed out that the stridhan of a woman is her absolute property and the husband has no interest in the same and the entrustment to him is just like something which he wife keeps in a Bank and can withdraw any amount whenever she likes without any hitch or hindrance and the husband cannot use the stridhan for his personal purposes unless he obtains the tacit consent of his wife. When the essential conditions of a partnership do not exist the mere act or factum of entrustment of stridhan would not constitute any co-ownership or legal partnership as defined under s.4 of the Partnership Act.

To sum up the position seems to be that a pure and simple entrustment of stridhan without creating any rights in the husband excepting putting the articles in his possession does not entitle him to use the same to the detriment of his wife without her consent. The husband has no justification for not returning the said articles as and when demanded by the wife nor can he burden her with loss-, of business by using the said property which was never intended by her while entrusting possession of stridhan.) On the allegations in the complaint, the husband is no more and no less than a pure and simple custodian acting on b-half of his wife and if he diverts the entrusted property elsewhere or for different purposes he takes a clear risk of prosecution under s.406 of the IPC. On a parity of reasoning, it is mainfest that the husband, being only a custodian of the stridhan of his wife, cannot be said to be in joint possession thereof and thus acquire a joint interest in the property.

For these reasons, the custody or entrustment of stridhan with the husband does not amount to a partnership in any sense of the term and therefore, we are unable to agree with view taken in Vinod Kumar's case as also with the opinion expressed by our Brother on the points arising in the case.

Another serious consequence as a result of the ratio of the full Bench decision in Vinod Kumar's case would be to render the

212

provisions of s. 406 IPC inapplicable and nugatory even if the husband has the audacity or the importunity of refusing to return the stridhan of his wife. Furthermore, we shall hereafter show that the view of the Full Bench is in direct contravention of a long course of decisions of this Court on the ingredients of s. 405 IPC. Before coming to this chapter, we would like to say a few things more about the judgment of the High Court which on deeper probe and careful scrutiny seems to be self-contradictory.

We are clearly of the opinion that the concept of stridhan property of a married woman becoming a joint property of both the spouses as soon as she enters her matrimonial home and continues to be so until she remains there or even if there is a break in the matrimonial alliance, is in direct contravention of Hindu Law of Sadayika which has been administered since more than a century by High Courts, Privy- Council as also this Court. By a pure and simple figment of the fertile imagination the Judges in Vinod Kumar's case seem to have rewritten the law of criminal breach of trust contained in ss. 405 and 406 IPC so as to carve out an imaginary exception to the application of the Penal Code.A more tragic consequence of the view taken by the High Court is that even if there is a break in the matrimonial alliance and the wife wants her husband to return her exclusive property and he refuses to return, even then the provisions of s. 406 IPC would not apply. It is an extreme travesty of justice for a court to say that whenever a married woman demands- her stridhan property from her husband she should be driven to the dilatory process of a civil court and her husband would be debarred from being prosecuted by a criminal court. By a strange and ingenious process of holding that such an act of a husband does not attract the provisions of the Penal Code, as the property being joint there is no question of the husband being a trustee or holding the same in a fiduciary capacity. Such a view, in our opinion, is not only contradictory but-what the High Court has said before regarding the applicability of s.27 of the Hindu Marriage Act and the nature of stridhan as referred to above-is also neither in consonance with logic and reason nor with the express provisions of the Penal Code and seems to us to be inspired by a spirit of male chauvininism so as to exclude the husband from criminal lability

merely because his wife has refused to live in her matrimonial

213

home. We are indeed surprised how could the High Court, functioning in a civilised and socialistic society such as ours, play havoc with judicial interpretation of an important branch of law.

We shall now show how the final view taken by the High Court is clearly contradictory to what it has observed before. In paragraphs 22A, 23 and 24 of the judgment, the High Court observes as follows:

"It must, therefore, be unreservedly stated that the law, as it stands today, visualises a complete and full ownership of her individual property by a Hindu wife and in this context the factum of marriage is of little or no relevance and she can own and possess property in the same manner as a Hindu male. Once it is held that a Hindu wife can own property in her own right, then it is purely a question of fact whether the dowry or the traditional presents given to her, were to be individually owned by her or had been gifted to the husband alone or jointly to the couple.- .. For instance jewellery meant for the personal wearing of the bride, wedding apparel made to her measures specifically, cash amounts put into a fixed deposit ill a bank expressly in her E name; are obvious examples of dowry raising the strongest, if not conclusive presumption, of her separate owner ship in these articles. Once it is found as a fact that these articles of dowry were so given to her individually and in her own right, then I am unable to see how the mere factum of marriage would alter any such property right and divest her of ownership either totally or partially."

In these paragraphs the High Court unequivocally and categorically expresses the view that a Hindu woman has complete and full ownership of her individual property and the factum of marriage is of no relevance to determine the nature of the property It also holds that articles like jewellery, wedding apparel and cash, etc., cannot alter any such property right. In view of this clear finding given by the High Court, how could it make a complete volte-face by holding that these very properties after marriage become joint property of both the spouses. The High Court has not realised that the theory or philosophy of matrimonial home 214

propounded by it stands directly contradicted by its own observations referred to above.

In paragraph 49 of the judgment, the High Court clearly finds that the mere use by the relations of the husband would not have the effect of passing the possession of the property to the Hindu undivided family and in this connection observes thus :-

Equally, the common use and enjoyment of certain articles of dowry and traditional presents, by the other members of a joint family with the leave and licence of a Hindu wife, cannot have the effect of extending the jointness Of control and custody of the couple to undefined and unreasonable limits. Consequently, there is no reason to assume that the mere user or enjoyment of the dowry by other members of the house-hold, would have the effect of passing the possession and control thereof jointly to the Hindu Undivided Family as such."

Thus, these observations run counter and are totally inconsistent and irreconcilable with the view taken by the High Court in paragraph 41 where it has observed thus: "In the light of the above it would be farcical to assume that despite the factum of a marriage and a common matrimonial home the two spouses would stand in a kind of a formal relationship where each is entrusted with or has been passed dominion over the exclusive property of the other..-....The matrimonial home so long as it subsist presumes a jointness of custody and possession by the spouses of their individual as also of their joint properties line.. The inevitable presumption during the existence or the imminent break up of the matrimonial home there fore is one of joint possession of the spouses which might perhaps be dislodged by the special terms of a written contract. However, to be precise this presumption of joint possession properties within the matrimonial home can subsist only as long- as the matrimonial home subsists or on the immediate break up thereof."

At other places the High Court has observed thus: 215

"47. In view of the above, it would be equally untenable to hold that either the desertion or the expulsion of one of the spouses from the matrimonial home would result in entrusting dominion over the property belonging to the other so as to bring the case within the ambit of this pre requisite under S.405, Indian Penal Code. The joint custody and possession once established would thereafter........ exclude either express entrustment or the passing of dominion over the property. It was rightly argued that if an irate husband or wife walks out from the matrimonial home in a huff, this cannot constitute an entrustment or dominion over the property to the other. Consequently, unless a special written agreement to the contrary can be established, the strongest presumption arises that during the existence and immediately after the crumbling of the matrimonial home, there was in essence, a joint possession and custody of the property of the spouses therein, including dowry and traditional presents, which would preclude the essentials of entrustment or dominion over the property which form the cornerstone of criminality under s.405, Indian Penal Code.

53. It cannot, therefore, be prim. facie presumed that these are exclusively the ownership of the wife or inevitably entrusted either to the husband or his close relations. As was noticed earlier, if an irate wife in a tantrums abandons the matrimonial home, such like property does not in the eye of law become entrusted to the parents-in-law or other close relations of the husband No such gullible presumption of entrustment or passing of the dominion of property can be raised in such a situation to come within the mischief of criminality for breach of trust. Entrustment or dominion over the property has to be unequivocally alleged and conclusively established by proof later." The High Court had itself rightly spelt out the legal propositions that the pure and traditional presents given to a bride in a Hindu wedding may be divided into three categories, viz.,

(a) property intended for exclusive use of the bride, e. g., her personal jewellery, wearing apparel, etc.

216

(b) articles of dowry which may be for common use and enjoyment in the matrimonial home, and

(c) articles given as presents to the husband or the parent-in -law and other members of his family. With regard to category (a) above, the High Court observed thus:

"Similarly as regards the first category of articles meant for the exclusive use of the bride she would retain her pristine ownership therein irrespective or her entry and presence in the matrimonial home or that of her parents in-law." The High Court thus accepts the well established rule of Hindu law of stridhan that that articles mentioned in category (a) are meant for the exclusive use of the bride and are her personal property. Unfortunately, however, with regard to category (c) while discussing the question of the rights of the bride to her exclusive property upon her entry in her matrimonial home, the High Court has wrongly applied what it had previously held with regard to category (a). In one breath the Judges say that the bride is entitled to retain her ownership irrespective of her entry and presence in the matrimonial home and in the other they come to the conclusion that the moment a married woman enters her matrimonial home, all her properties, including her exclusive property, become a joint property by a fiction of being placed in the custody of her husband or his relations. While we agree with the first part of the categories, as extracted above, we find it difficult to accept the other propositions adumbrated at a later stage of the judgment which have been fully discussed by us. We fail to understand how the High Court while finding that joint enjoyment does not divest a Hindu wife of her exclusive ownership still chose to treat it a joint property of the two spouses by the mere factum of joint user. The two views expressed by the High Court stand contradicted by its own findings and are wholly understandable. Thus, a detailed analysis of the judgment of the Punjab & Haryana High Court in Vinod Kumar's case (supra) appears to us to be a mass of confusion and lacks both clarity and coherence. We are, therefore, unable 217

to uphold or support the view of the High Court that upon entering the matrimonial home the ownership of stridhan property becomes joint with her husband or his relations- To this extent, therefore, we overrule this decision and hold that with regard to the stridhan property of a married woman, even if it is placed in the custody of her husband or in-laws they would be deemed to be trustees and bound to return the same if and when demanded by her. The Supreme Court in a large number of cases has held that the fundamental core of the offence of criminal breach of trust. is that a property must be entrusted and the dominion of the property should be given to the trustee. In the present case, all these conditions, even according to the findings of the Court though not its conclusion, are clearly established. That the view of the High Court is absolutely wrong would be clear from a number of authorities, some of which we would like to discuss here. In Chelloor Manaklal Narayan Ittiravi Nambudiri v. State of Travancore(1) this Court made the following observations:

"As laid down in S. 385, Cochin Penal Code (corresponding to S. 405, Indian Penal Code) to constitute an offence of criminal breach of trust it is essential that the prosecution must prove first of all that the accused was entrusted with some property or with any dominion or power over it-It follows almost axiomatically from this definition that the ownership or beneficial interest in the property in respect of which criminal breach of trust is alleged to have been committed, must be in some person other than the accused and the latter must hold it on account of some person or in some way for his benefit."

In Jaswantrai Manilal Akhaney v., State of Bombay(2) Sinha, J. (as he then was) observed thus: "For an offence under section 409, Indian Penal Code, the first essential ingredient to be proved is that the property was entrusted- - But when section 405 which defines "criminal breach of trust speaks of a person being

(1) AIR 1953 SC 478.

(2) [1956] S.C.R. 483.

218

in any manner entrusted with property, it does not contemplate the creation of a trust with all the technicalities of trust. It contemplates the creation of a relationship whereby the owner of property makes it over to another person to be retained by him until a certain contingency arises or to be disposed of by him on the happening of a certain events."

In Akharbhai Nazorali v. Md. Hussain Bhai(1) the Madhya Pradesh High Court made the following observations: "It may be that the deduction and retention of the employees' contribution is a trust created by virtue of that very fact, or by virtue of a provision in statute or statutory rule. But even apart from the latter, the mere fact of telling the employees that it is their contribution to the provident fund scheme and then making a deduction or recovery and retaining it, constitutes the offence of criminal breach of trust. This is so obvious that nothing more need be said about it."

These observations were fully endorsed and approved by this Court in Harihar Prasad Dubey v. Tulsi Das Mundhra & Ors.(2 where the following observations were made: "This, in our opinion, is a correct statement of the position and we also agree with the learned Judge of the Madhya Pradesh High Court that "this so obvious that nothing more need be said about it We, therefore, think that the impugned order quashing the charge against the respondents is obviously wrong." In Basudeb Patra v. Kanai Lal Haldar(3) the Calcutta High Court observed thus:

"Whereas the illustration to s. 405 show equally clearly that the property comes into.

(1) AIR 1961 M. P. 37:

(2) AIR 1981 SC 92.

(3) AIR 1949 Calcutta 207,

219

the possession of the accused either by an express entrustment or by some process placing the accused in a position of trust..-.-On the facts of the present case, which, as I have said, are not open to question at this stage, it is quite clear that the ornaments were handed over to the petitioner by the beneficial owner in the confidence that they would be returned to the beneficial owner in due time after having been used for the purpose for which they were handed over. If this is not an entrustment, if is impossible to conceive what can be an entrustment."

(Emphasis ours)

This ratio was fully approved by this Court in Velji Raghavji Patel v. State of Maharashtra(1) where the following observation were made:

"In order to establish " entrustment of dominion" over property to an accused person the mere existence of that person's dominion over property is not enough. It must be further shown that his dominion was the result of entrustment. Therefore, as rightly pointed out by Harris, C.J. the prosecution must establish that dominion over the assets or a particular asset of the partnership was by a special agreement between the parties, entrusted to the accused person." In the case of State of Gujrat v. Jaswantlal Nathalal,(2) Hegde, J., speaking for the Court, observed thus:

"The expression 'entrustment' carries with it the implication that the person handing over any property or on whose behalf that property is handed over to another, continues to be its owner. Further the person handing over the property must have confidence in the person taking the property so as to create a fiduciary relationship between them."

In Sushil Kumar Gupta v. Joy Shanker Bhattacharjee(3) this Court observed thus:

(1) AIR 1965 SC 1433.

(2) [1968] 2 SCR 408.

(3) AIR 1971 SC 1543.

220

"The offence of criminal breach of trust is committed when a person who is entrusted in any manner with property or with dominion over it, dishonestly misappropriates it or converts it to his own use.. The appellant's manner of dealing with the money entrusted to his custody clearly constitutes criminal breach of trust."

In the case of Superintendent & Remembrancer of Legal Affairs, West Bengal v. S.K. Roy (1) this Court held that for 'entrustment' two things are necessary, viz., (l) the entrustment may arise in "any manner" whether or not it is fraudulent, and (2) the accused must have acquisition or dominion over the property.

In Bhai Sher Jang Singh & Anr. v. Smt. Virinder Kaur (supra) the Punjab & Haryana High Court observed thus: It might be that some of the articles which were presented to her are for the use of both the spouses but the ornaments and things of the like nature are certainly meant for her and her alone. When she makes an allegation in the complaint that either her husband or her parents-in-law had converted to their own use the ornaments forming the part of her stridhan which she had entrusted to them, the Court has to give legal effect to such allegation and to assume that such ornaments had been made the subject matter of criminal breach of trust. It is settled law that even in a criminal complaint the complainant is under no obligation to plead the legal effect of the allegations made. All that is required is that the facts constituting a complaint should be specifically mentioned so that the Court may be able to perform its duty of punishing the accused under the appropriate provision of law if such allegations are made out. Further more, in a case like this a complaint cannot be quashed without giving the aggrieve wife an opportunity of proving that the ornaments had been given to her at the time of her marriage for her use only." (Emphasis supplied)

We fully endorse this decision and hold that it lays down the correct law on the subject.

(1) [1974] 4 S.C.C. 230.

221

There is a judgment of the Allahabad High Court which more or less takes the same view as the Punjab & Haryana High Court in Vinod Kumar's case (supra). In Criminal Misc. Case No. 676 of 1981 (connected with) Criminal Misc. Case No. 2753 of 1981, Kailash Nath Agarwal & Ors. v. Prem Pal Agarwal & Anr., (decided on 22.12.1983), the Allahabad High Court, out of the three categories laid down by Punjab & Haryana High Court in Vinod Kumar's case, accepted only the third category, viz., articles which constitute the individual property of the person for whose use it was given, and held that the rest of the property falling under categories (a) and (b) would be property exclusively meant for the use of the bride and once it was brought to the family home, the possession would be joint unless by an express written agreement there was an entrustment of the property of the bride to other members of the family. The Allahabad High Court thus also accepts the concept of the property being a joint property in the matrimonial home. By and large this decision toes the line of the view taken by the Punjab and Haryana High Court in Vinod Kumar's case. Furthermore, the High Court has gravely erred in holding that the property could only be claimed by filing a properly constituted civil suit or in accordance with the provisions of the Dowry Prohibition Act or the Hindu Marriage Act as the case may be. This proposition, in our opinion, is wholly incorrect as conceded even by the Punjab , & Haryana High Court in Vinod Kumar's case.

There is an earlier decision of the Punjab & Haryana High Court which clearly holds that where there is a clear allegation of entrustment by the wife against the husband, he could be prosecuted by a criminal court on a complaint filed by the wife. In this connection, the Court in Avtar Singh & Anr. v. Kirpal Kaur Criminal Misc. No.2144-M of 1979 and Criminal Misc.No.2145 of 1979, decided on 16 8.79) made the following observations:

"In my opinion, where certain thing is lying in trust with a person, offence of dishonest misappropriation would be committed on a date the demand for return of the entrusted articles is made and the same is declined...According to the complaint, the first demand for the return of the articles was made on January 27, 1976 and it was that date when the demand was declined. Hence, the offence of misappropriation of the dowry articles lying in trust was committed on January 27, 1976."

222

We find ourselves in entire agreement with this decision and hold that this was correctly decided. This Court has pointed out more than once that the High Court should very sparingly exercise its discretion under s. 482 Cr. P.C. In L.V. Jadhav v. Shankarrao Abasaheb Pawar & Ors.(l) (to which two of us were a party), this Court made the following observations:

"The High Court, we cannot refrain from observing, might well have refused to invoke its inherent powers at the very threshold in order to quash the proceedings, for these powers are meant to be exercised sparingly and with circumspection when there is reason to believe that the process of law is being misused to harass a citizen."

In Smt. Nagawwa v. Veeranna Shivalingappa Konjalgi & Ors.(2) this Court observed as follows :- "Thus, it may be safely held that in the following cases an order of the magistrate issuing process against the accused can be quashed or set aside: (1) Where the allegations made in the complaint or the statements of the witnesses recorded in support of the same taken at their face value make out absolutely no case against the accused or the complaint does not disclose the essential ingredients of an offence which is alleged against the accused;

(2) Where the allegations made in the complaint are patently absurd and inherently improbable so that no prudent person can ever reach a conclusion that there is sufficient ground for proceeding against the accused;

(3) where the discretion exercised by the Magistrate in suing process is capricious and arbitrary having been either on no evidence or on materials which are wholly irrelevant or inadmissible; and (1) AIR [1983]SC 1219.

(2) [1976] Supp. SCR123

223

(4) where the complaint suffers from fundamental legal defects, such as, want of section, or absence of a complaint by legally competent authority and the like.

The cases mentioned by us are purely illustrative and pro vide sufficient guidelines to indicate contingencies where the High Court can quash proceedings." B

The same principles would apply mutatis mutandis to a criminal complaint.

We now come to the question as to whether or not a clear allegation of entrustment and misappropriation of properties was made by the appellant in her complaint and, if so, was the High Court justified in quashing the complaint at that stage. It is well settled by a long course of this Court that for the purpose of exercising its power under s. 482 Cr.P.C. to quash a FIR or a complaint the High Court would have to proceed entirely on the basis of the allegations made in the complaint or the documents accompanying the same per se. It has no jurisdiction to examine the correctness or otherwise of the allegations. In case no offence is committed on the allegation and the ingredients of s.405 & 406, I.P.C. are not made out, the High Court would be justified in quashing the proceedings. In the present case, we shall show that the allegations are both clear, specific and unambiguous and, therefore, the complainant should have been given a chance to prove her case. It is, of course, open to the accused at the trial to take whatever defence that were open to him but that stage had not yet come and therefore, the High Court was totally ill-advised to speculate on the merits of the case at that stage and quash the proceedings. We have narrated the facts in detail in the earlier part of our judgment but we might again, even at the risk of repetition, indicate the bare facts which prima facie make out a clear case under s.406, IPC against the accused. The important portions of the complaint may be spelt out thus:

(1) that all the accused attended the marriage of the appellant with the respondent and demanded dowry from the parents of the appellant in consideration of the marriage.

(2) that the parents of the appellant spent Rs,75,000 on the marriage and dowry articles worth Rs.60,000 224

(inclusive of jewellery, wearing apparel, etc.) were given and entrusted to accused Nos.1 to 6 at the time of the Doli on 5.2.72,

(3) that the articles entrusted to the accused were meant for the exclusive use of the appellant, (4) that the dowry articles were never given by the accused to the appellant even for her use and possession of the same was illegally, dishonestly and mala fidely retained by the accused in order to obtain a wrongful gain to themselves and wrongful loss to the appellant,

(5) that on 11.12.1980 in the morning, the accused brought the appellant to Ludhiana in three clothes and refused to give the entrusted articles which were the stridhan of the appellant.

Taking all the allegations made above, by no stretch of imagination can it be said that the allegations do not prima facie amount to an offence of criminal breach of trust against the respondent. Thus, there can be no room for doubt that all the facts stated in the complaint constitute an offence under s. 406 IPC and the appellant cannot be denied the right to prove her case at the trial by per-empting it at the very behest by the order passed by the High Court. We therefore, overrule the decisions of the Punjab & Haryana High Court in Vinod Kumar's case. By way of post-scriptt we might add that we are indeed amazed to find that so deeply drowned and inherently engrossed are some of the High Courts in the concept of matrimonial home qua the stridhan property of a married women that they simply refuse to believe that such properties are meant for the exclusive use of the wife and could also be legally entrusted to the husband or his relatives. Thus, if the husband or his relatives misappropriate the same and refuse to hand it over to the wife and convert them to their own use and even though these facts are clearly alleged in a complaint for an offence under s. 405/406 I.P.C., some courts take the complaint is not maintainable. Thus, even when clear and specific allegations are made in the complaint

225

that such properties were entrusted to the husband, they refuse to believe these hard facts and brush them aside on the ground that they are vague. The allegations of the complainant in this appeal and the appeal before the Allahabad and the Punjab & Haryana High Court show that it is not so but is a pure figment of the High Court's imagination as a result of which the High Court completely shut their eyes to the fact that the husband could also be guilty under s. 405/406 I P.C. in view of the clear allegations made in the complaint. In other words, the High Courts simply refuse to believe that there can be any such entrustment and even if it is so no offence is committed. Such an approach amounts to a serious distortion of the criminal law, resulting in perpetrating grave and substantial miscarriage of justice to the wife at the hands of the High Courts. We cannot countenance such a wrong and perverse approach.

For the reasons given above, we are satisfied that as the complaint prima facie disclosed an offence of criminal breach of trust as defined in s. 405/406 of the Indian Penal Code the High Court was not justified in quashing`the complaint. We, therefore, allow this appeal, set aside the judgment of the High Court and restore the complaint filed by the appellant and direct that the accused may be summoned, if not already summoned, and put on trial in accordance with law.

VARADARAJAN, J. This criminal appeal by special leave is directed against the judgment of a learned Single Judge of the Punjab and Haryana High Court in Criminal Misc. Case No.4876 of 1981.

The appellant, Pratibha Rani is the estranged wife of the first respondent Suraj Kumar who is the brother of the second respondent Krishan Lal. One Rattan Chand is the father of respondents 1 and 2 and two others Chander Kumar and Vishwinder Kumar. One Jugal Kumar is the brother-in-law of the first respondent.

The appellant filed a criminal complaint for an offence under s.406 I.P.C. against her husband and his father and brothers and brother-in-law mentioned above in the Court of the Additional Chief Judicial Magistrate, Ludhiana, alleging that she was married to the first respondent at Ludhiana on

4. 2. 1972 according to the Hindu rites and customs. The material averments in the complaint

226

are these: The aforesaid persons, namely, father, brother and brother-in-law of the first respondent attended the marriage and demanded dowry from the appellant's parents as consideration for the marriage. Accordingly, dowry articles mentioned in the list appended to the complaint, worth Rs. 60,000, in the form of golden articles, clothes and other valuables were given and entrusted to the respondents and four others mentioned in the complaint at Ludhiana time of 'doli' on 5.2.1972 in the presence of Kapur Chand Jain and six others. The six respondents in the complaint started teasing, harassing and beating the appellant and they kept her without even food to extract more money from her parents. They turned out the appellant with her children in the beginning of 1977. After a great deal of persuasion and intervention by Panchayatdars, respondent 1 came to Ludhiana and took the appellant to his house after giving an undertaking in writing on 21. 6. 1977 not to misbehave with and maltreat the appellant her children. But after some time all the respondents in the complaint started maltreating the appellant and misbehaving with her. The articles mentioned in the list were never given by the respondents in the complaint to the appellant for her use but were retained by them illegally and with the dishonest intention of causing wrongful gain to themselves and wrongful loss to the appellant. The respondents in the complaint brought the appellant to Ludhiana at 4.30 a.m. On 11.12.1980 and left her near Kailash Cinema Chowk. They refused to give the articles mentioned in the list which are the stridhan of the appellant to her. When the appellant's husband and his brother, Vishwinder Kumar, respondents 1 and 5 in the complaint, came to Ludhiana on 10.2.1981 to attend the proceeding started by the appellant under s. 125 Cr. P.C., her parents persuaded them to return the articles entrusted to them at the time of the marriage but they flatly refused to comply with that demand. The articles have not been returned in spite of service of notice dated 17. 12. 1981 on the first respondent. Thus the respondents in the complaint have dishonestly converted the articles belonging to the appellant for their use in violation of the direction of the appellant's parents given at the time of the marriage to give the articles for the appellant's use. The respondents in this appeal filed Criminal Misc. Case No.4876 of 1981 in the Punjab and Haryana High Court under s.482 of the Code of Criminal procedure for quashing the criminal

227

Proceedings and the complaint taken on file by the Additional Chief Judicial Magistrate, Ludhiana under s. 406 I.P.C. and his order summoning them.

Sukhdev Singh Kang, J. before whom the matter came up in the High Court relied strongly upon the observations made by a Full Bench of that High Court in Vinod Kumar Sethi & Ors. v. State of PunJab and Ors.(l) and has observed in his judgment that the mere handing over of the articles of dowry of stridhana to the husband and other relations at the time of the marriage does not constitute entrustment in the sense of the word used in ss. 405 and 406 I P.C. and that it does not amount to passing of dominion over those articles to them. The learned Judge has observed that there can be such an entrustment only by a subsequent conscious act of volition ` and that in the absence of such an act any allegations of breach of trust between the husband and wife cannot constitute an offence under s.406 I.P.C. The learned Judge has further observed that between the husband and wife there is always a jointness of control and possession of the properties of the spouse within the matrimonial home and that it goes against the very concept of entrustment of his or her property by one spouse to the other. In this view, he allowed the petition and quashed the proceeding arising out of the appellant's complaint, observing that the allegations in the appellant's complaint are similar to the one in Vinod Kumar's case (supra) and that this case is fully covered by the ratio in that decision.

The appellant has, therefore, come to this Court in appeal by special leave, impleading the petitioners before the High Court, who are only two out of the six respondents in the complaint, as respondents in this appeal. In a petition under s.482 Cr.P.C. for quashing a criminal complaint, the allegations made in the complaint have to be taken to be correct in order to find out whether they constitute the various ingredient of the offence alleged. In Nagawa Veernna Shivalingappa Konjalgi & Ors ) illustrations have been given of cases in which it may be safely held that an order of a Magistrate issuing process against an accused can be quashed or set aside. They are: (1) AIR 1982 Punjab 372.

(2) [1976] Suppl S.C.R. 123

228

(1) Where the allegations made in the complaint or the statements of the witnesses recorded in support of the same, taken at their face value, make out absolutely no case against the accused or the complaint does not disclose the essential ingredients of an offence which is alleged against the accused; (2) Where the allegations made in the complaint are palpably absurd and inherently improbable so that no prudent person can ever reach a conclusion that there is sufficient ground for proceeding against the accused;

(3) Where the discretion exercised by the Magistrate in issuing process is capricious and arbitrary having been based either on no evidence or on materials which are wholly irrelevant or inadmissible; and

(4) Where the complaint suffers from fundamental legal defects such as want of sanction, or absence of a complaint by a legally competent authority and the like."

Article 126 in Mulla's Hindu Law, Fifteenth Edition, describing what constitutes Stridhana reads: - "property given or bequeathed to a Hindu female whether during maidenhood, coverture or widowhood by her parents and their relation or by her husband and his relations is stridhana according to all schools except that the Dayabhaga does not recognise immovable property given or bequeathed by husband to his wife as stridhana."

Section 2 of the Dowry prohibition Act, 1961 defines "dowry" as meaning:

"any property or valuable security given or agreed to be given either directly or indirectly-(a) by one party to a marriage to the other party to the marriage, or (b) by the parents of either party to the marriage, or by any other person to either party to the marriage or to any other person at or before of after the marriage in connection with the marriage of the said parties but does not include dower or mahr in the case of person to whom the Muslim personal law (Shariat) applies."

229

In the present complaint of the wife against the husband and , his three brothers, father and brother-in-law, it is alleged that the marriage was performed at Ludhiana on 4.2.1972 according to Hindu rites and customs and that the father and three brothers and the brother-in law of the husband attended the marriage and demanded dowry from the wife's parents as consideration for the marriage and that accordingly dowry articles worth Rs.60,000, mentioned in the list attached to the complaint, consisting of gold articles, clothes and other valuables were given and entrusted to the husband and the other five respondents in the complaint, at the time of the 'doli' at Ludhiana on 5.2. 1972 in the presence of Kapur Chand Jain and six other persons. For the purpose of the petition under s.482 Cr.P.C. those articles must be prima facie considered to be dowry or stridhana of the appellant-wife.

In Velji Raghavjl Patel v. State of Maharashtra,(1) it is observed:

"Upon the plain reading of s.405, I.P.C. it is obvious that before a person can be said to have committed criminal breach of trust it must be established that he was either entrusted with or entrusted with dominion over property which he is said to have converted to his own use or disposed of in violation of any direction of law etc. Every partner has dominion over property by reason of the fact that he is a partner. This is a kind of dominion which every owner of property has over his property. But it is not dominion of the kind which satisfies the requirements of s. 405. In order to establish "entrustment of dominion" over property to an accused person the mere existence of that person's dominion over property is not enough. It must be further shown that his dominion was the result of entrustment. Therefore, as rightly pointed out by Harris C.J., the prosecution must establish that dominion over the assets or a particular asset of the partnership was by a special agreement between the parties, entrusted to the accused person. If in the absence of such a a special agreement a partner receives money belonging to the partnership he cannot be said to have received it in a

(1) [1965] 2 S C.R. 429

230

fiduciary capacity or in other words cannot be held to have been "entrusted" with dominion over partnership properties."

In State of Gujarat v. Jaswantlal Nathalal(1) it is observed:

"Before there can be any entrustment there must be a trust meaning thereby an obligation annexed to the owner ship of property and a confidence reposed in and accepted by the owner or declared and accepted by him for the benefit of another or of another and the owner. But that does not mean that such an entrustment need conform to all the technicalities of the law of trust - seeJaswantrai Manilal Akhaney v. State of Bombay [1956] SCR 483, 498-500. The expression 'entrustment' carries with it the implication that the person handing over any property or on whose behalf that property is handed over to anther, continues to be its owner. Further the person handing over the property must have confidence in the person taking the property so as to create a fiduciary relationship between them." In Sushil Kumar Gupta v. Joy Shankar Bhattacharyya(2), it is observed:

"The offence of criminal breach of trust is committed when a person who is entrusted in any manner with property or with dominion over it, dishonestly misappropriates it, or converts it to his own use, or dishonestly uses it or disposes it of, in violation of any direction of law prescribing the mode in which the trust is to be discharged, or of any lawful contract, express or implied, made by him touching such discharge, or wilfully suffers any other person so to do."

In Superintendent Remembrancer of Legal Affairs, West Bengal v. S.K. Roy(8), it is observed:

"There are, however, two distinct parts involved in the commission of the offence of criminal breach of trust. The first consists of the creation of an obligation in rela

(1) [1968] 2 SCR 408.

(2) [1970] 3 SCR, 770.

(3) [1974] 4 SCC,230.

231

tion to the property over which dominion or control is acquired by the accused. The second is a misappropriation or dealing with the property dishonestly and contrary to the terms of the obligation created.

The most important ingredient of an offence under s. 406, which is alleged by the wife against her husband, his three brothers, father and brother-in-law in her complaint in the present case is the entrustment of the dowry articles to the respondent in the complaint and ,their dishonest conversion thereof to their own use. There is no doubt an allegation in the complaint that these articles were given and entrusted to the respondents in the complaint at Ludhiana at the time of doll on 5-2-1972. Apart from the husband the other respondents in the complaint, as already stated, are his father, three brothers and brother-in-law. The articles were given for the use of the wife- If so, could there be entrustment of the articles to such a number of diverse persons? In the background of what usually happens in Hindu marriages namely, placing of the articles presented to the bride in the presence of the elders and others assembled for the occasion and removal thereof after the function is over it has to be seen whether the allegation made in the complaint amounts to entrustment as required by law to make out an offence under s. 406 l.P.C. This question has been considered in detail by a Full Bench of the Punjab and Haryana High Court in Vinod Kumar's case (supra) after an analysis of several decision relating to the question. The learned Single Judge who has quashed the complaint in the present case on a petition of the husband and one of his brothers has heavily relied upon that Full Bench decision of his Court. What runs through the judgment of the learned Judges in that case is the concern of the Court for the peaceful and harmonious relationship between the spouses in a matrimonial home and a careful consideration of the question whether the ingredient of entrustment" exists in such cases. Therefore, it is necessary to note what has been observed in some of the paragraphs of the judgment to that case. The learned Chief justice speaking for the Bench has observed: "21.. The present set of cases presents a sad spectacle of a house divided against itself, not merely in the biblical but in the literal sense, where wives are ranged against their husbands in acrimonious criminal prosecu-

232

tions. The challenge on behalf of the husbands and their relations is focussed basically against the charge of breach of trust under Section 406 of the Indian Penal Code, levelled against them. Now the core of the argument on behalf of the petitioners is that the very concept of any entrustment or passing dominion over her property by the wife to the husband does not arise at all so long as the marriage subsists. The contention is that the very nature of the conjugal relationship itself would negative any such stand. On this premise it is contended that the basic pre- requisite of the entrustment of property or dominion over property being lacking and non-existent, no offence under Section 406, Indian renal Code, can possibly be made out. Therefore, it was argued that even accepting the first information reports as they do not and indeed cannot disclose a cognizable offence under Section 406. The petitioners, therefore, seek the quashing of the proceedings - forthwith rather than being obliged to go through the tortuous mill of a police investigation or the consequent criminal trial." "25. Now apart from the principle, the most ancient texts of Hindu Law have always been categoric that dowry, as commonly understood, was stridhana and thus in the exclusive ownership of the bride." "26. Now once it is so held that articles of dowry and traditional presents given at the wedding are owned by the bride individually in her own right, then one fails to see how by the mere fact of her bringing the same into her husband's or parents-in-law's household, would forth with divest her of the ownership thereof. Separate and individual right to property of the wife therein cannot vanish into thin air the moment the threshold of the matrimonial home is crossed. To say that at that point - of time she would cease to own such property altogether and the title therein would pass to her husband or in any case she would lose half of her right therein and become merely a joint owner of the same, with the family of her husband, does not appear to me as even remotely warranted either by the statute, principles or logic. No such marriage hazard against the wife can be implied in law.

233

Once she owns property exclusively, she would continue to hold and own it as such despite marriage and coverture and the factum of entering the matrimonial home.. "

"35. To conclude on this aspect, I find nothing in the codification of Hindu Law which in any way abolishes the concept of stridhana or the right of a Hindu wife to exclusive individual ownership. Indeed the resultant effect of such enactments is to put the Hindu female wholly at par with the Hindu male, if not at a higher pedestal with regard to individual ownership of the property."

40. Now having held as above that Hindu wife can exclusively own and hold property including her dowry and traditional presents given at the wedding, the decks are cleared for tackling the core question posed at the very outset. What indeed is the true legal relationship of the husband and wife qua the property individually owned by each within the four walls of the matrimonial home? Does the wife stand entrusted with the property belonging to her husband individually and vice versa the husband stands entrusted with such property vesting in the exclusive ownership of the wife? It is the answer to this question which in essence would determine the attraction and applicability of Section 405, I.P.C betwixt the spouses.."

"41. It bears 'repetition that the question herein has to be examined against the backdrop of the matrimonial home. What truly is the concept and essence thereof had come up for exhaustive consideration earlier before a Full Bench in Kailash Vati v. Ayodhia Parkash, ILR (1977) 1 Punj. & Har. 642 in the context of Hindu Law itself. It is, therefore, apt to refer to the authoritative enunciation therein:-

"To my mind, the idea of the matrimonial home appears to lie at the very centre of the concept of marriage in all civilised societies. It is indeed around it that generally the marriage tie revolves. The home epitomizes the finer nuances of the marital status. The bundle of indefinable rights and duties which

234

bind the husband and the wife can perhaps be best understood only in the context of their living together in the marital home The significance of the conjugal home in the marriage tie is indeed so patent that it would perhaps be wasteful to elaborate the 8 same at any great length. Indeed, the marital status and the conjugal home have been almost used as interchangeable terms." and

"To summarise, I have attempted to show by reference to Anglo-American Jurisprudence that the a concept of the marital home lies at the very centre of the idea of marriage in all civilised societies. Perhaps from primeval times when human beings lived sheltered in subterranean caves to the modern day when many live perched in flats in high rise apartments within the megapolis, the husband and the wife have always hankered for a place which may be their very own and which they may call a home. The innumerable mutual obligations and rights which stem from the living together of man and wife are undoubtedly beyond any precise definition and stand epitomized by the concept of the matrimonial home."

In the light of the above it would be farcical to assume that despite the factum of a marriage and a common matrimonial home the two Spouses would stand in a kind of a formal relationship where each is entrusted with or has been passed dominion over the exclusive property of the other. Rather it appears to me that the conjugal relationship and the existence of a matrimonial home automatically obviates any such hyper-technicalities of an entrustment or dominion over property. It seems inapt to conceive the relationship as a day-to-day entrustment of the property of the husband to the custody of the wife or vice versa of the property of the wife to the husband. The matrimonial home so long as it subsists presumes a jointness of custody and possession by the spouses of their individual as also of their joint properties which can not be divided by any metaphorical line. In a homely metaphor in the context of the modern commercialised world it has been said that the marriage relationship is not one of

235

"I and You limited" but that of "We limited". Whilst the law undoubtedly now clearly recognises the individual ownership of property by the husband and wife, the necessary assumption in law, therefore, would be that during the existence or even the imminent break up the matrimonial home the concept of jaintness of possession therein seems to be a paramount one. The inevitable presumption during the existence or the imminent break up of the matrimonial home therefore is one of joint possession of the spouses which might perhaps be dislodged by the special terms of a written contract. However, to be precise this presumption of joint possession of properties within the matrimonial home can subsist only as long as the matrimonial home subsists or on the immediate break up thereof." "42-43. The aforesaid position seems to be well borne out by a homely example which was rightly advanced by Mr. Bhandare on behalf of the petitioners. It was submitted that where a husband entrusts a specific amount to a wife for paying the school fees of their children but in a shopping spree she converts the same into sarees for herself, would she thereby become liable to breach of trust under Section 406, Indian Penal Code? The answer would obviously appear to be in the negative. Similarly where a husband misuses or even appropriates any property exclusively belonging to his wife within the matrimonial home he hardly comes within the ambit of criminality under Section 406, Indian Penal Code. Usually if not invariably where the husband is the bread winner he brings home the month's wages and bands them over to the wife to be spent on the family. Would it be possible to say that if she use the same for herself and even against the consent of her husband she would be committing a criminal breach of trust? Obviously the answer would appear to be in the negative."

"44. One may now turn precisely to the language of the Code itself. Sec. 405 is in the following terms:- 236

"405. Criminal Breach of trust: Whoever being in any manner entrusted with property, or with any dominion over property, dishonestly

misappropriates or converts to his own use that property, or dishonestly uses or disposes of that property in violation of any direction of law prescribing the mode in, which such trust is to be discharged or of any legal contract, express or implied, which he has made touching the discharge of such trust, or wilfully suffers any other Person so to do, commits criminal breach of trust."

It is well-setted that from a legal contract, or violation of direction of law, the entrustment of property or dominion over property are the per- requisites for the applicability of the aforesaid provision. Once it is held as above, that property within the matrimonial home is in the joint possession and custody (despite rights of the individual ownership therein) then these very per-requisites of entrustment or dominion over property cannot be easily satisfied betwixt the spouses inter se. It is indeed well-settled that the very concept of the jointness of possession and custody would rule out the entrustment or dominion over property betwixt such joint custodians. In line with the concept of joint ownership where the possession of one joint owner is deemed to be the possession of all, the analogy,is to be extended that existence of the property within the matrimonial home rises a presumption that both the husband the wife are in possession thereof jointly and not that each one has entrusted his exclusive property to the custody of other. Subscribing to the latter view would be both overly hypertechnical and subversive of the very concept of marriage, the matrimonial home and the inevitable mutual trust which conjugality necessarily involves."

"45. It is obviously because of the afore said legal position and this inarticulate peremise underlying the same that the learned counsel for the State and the complainants were unable to cite even a single case of conviction for criminal breach of trust betwixt husband and wife. Even when

237

pointedly asked, counsel conceded that despite the diligent research neither under the Indian Penal Code, nor under the analogous provisions of English law could they lay their hands for over a century and a half on any case where such a conviction had been upheld. This paucity, rather the total absence of precedent, indirectly buttresses the view I have expressed above on principle and the statutory provisions. An analogy in their context may well be drawn from the Law of Partnership. However, at the very outset I would notice that the position is not identical because partnership envisages a joint or co-ownership of partnership property whereas in a conjugal relationship, as shown above, the spouses may well be the individual and exclusive owners of their respective properties. Nevertheless a marked similarity therein is that in partnership, co-ownership necessarily connotes a jointness of possession of partnership properties whilst the same position inheres in the matrimonial home where the spouses are deemed to be jointly in possession and custody. Now, barring some ancient notes of discordance, it seems to be now well accepted that a partner cannot be held guilty of criminal breach of trust qua partnership property except by virtue of a special agreement either written or conclusively established. This had always been so in English law until it was specifically and altered by Statute 31 and 32 Victoria c. 116 and it is now governed by the special provisions of the same and subsequent legislation. In India, however, in the absence of any statutory change, the legal position would continue to be the same. This came up for pointed consideration before a Full Bench of five Judges in Bhuban Mohan Das v. Surendra Mohan Das, AIR 1951 Cal, 69. The relief sought therein of quashing the proceedings under S. 406, Indian Penal Code, betwixt partners, was granted whilst holding that a charge under S. 406, Indian Penal Code cannot be framed against a person who, according to the complainant, is a partner with him and is accused of the offence in respect of property belonging to them as partners. P.B. Mukharji, J. in his concurring judgment observed as under (Para 46) : "The question here is of much broader application and of a more fundamental nature. Its fundamen- 238

tal nature is this that the very conception of partner ship precludes possibility of entrustment or dominion of the partnership property by one partner as against the other and, therefore, precludes any possible operation of the crime under Section 406 Penal Code, of criminal breach of trust by one partner against the other in respect of the partnership property." The aforesaid view has been expressly referred to and approved by their Lordships in Velji Raghavji v. State of Maharashtra,(1) with the following added observations (at pp. 1435-36) :-

"... Every partner has dominion over property by reason of the fact that he is a partner. This is a kind of dominion which every owner of property has over his property. out it is not dominion of this kind which satisfies the requirements of S.405. In order to establish `entrustment of dominion' over property to an accused person the mere existence of that person's dominion over property is not enough. It must be further shown that his dominion was the result of entrustment. Therefore, as rightly pointed out by Harris, C.J., the prosecution must establish that dominion over the assets or a particular asset of the partnership was, by a special agreement between the parties entrusted to the accused person. If in the absence of such a special agreement partner receives money belonging to the partnership he cannot be said to have received it in a fiduciary capacity or in other words cannot be held to have been 'entrusted' with dominion over partnership properties. "

If that is so in the partnership relation it appears to me that it would be more so in the conjugal relationship with regard to the property within the matrimonial home." "46..... The nature, character and the incident of property within the matrimonial home, so long as the marriage subsists, seem to be such that except by a special written agreement, no entrustment or dominion etc. Of the individual property of the spouses to each other can b e presumed. Equally, herein the specific and ascertainable

(I) A.T.R. 1965 S.C. 1433

239

property of each spouse within the matrimonial home can , be so equivocal and problematic as to oust the requisite mens rea with consequent criminality with regard thereto until the title to such property is clearly and specifically established. If the civil remedy seems to be adequate betwixt partners, during the subsistence of partnership there is no reason why it would not equally be so betwixt spouses in an existing matrimonial home during the subsistence of the conjugal relationship. As already referred to, apart from the civil remedy under the general law, added provisions exist in this context under S.27 of the Hindu Marriage Act buttressed by the procedural provisions of 0.32-A of the Code of Civil Procedure." "47. In view of the above, it would be equally untenable to hold that either the desertion or the expulsion one of the spouses from the matrimonial home would result in entrusting dominion over the property belonging to the other so as to bring the case within the ambit of this pre-requisite under S.405, Indian Penal Code. The joint custody and possession once established would thereafter exclude either express entrustment or the passing of dominion over the property. It was rightly argued that if an irate husband or wife walks out from the matrimonial home in a huff, this cannot constitute an entrustment or dominion over the property to the other. Consequently, unless a special written agreement to the contrary can be established, the strongest presumption arises that during the existence and immediately after the crumbling of the matrimonial home, there was in essence, a joint possession and custody of the property of the spouses therein, including dowry and traditional presents, which would preclude the essentials entrustment of dominion over the property which form the corner-stone of criminality under S.405, Indian Penal Code."

"49. Equally the common use and enjoyment of certain articles of dowry and traditional presents, by the other members of a joint family with the leave and licence of a Hindu wife, cannot have the effect of extending the jointness of control and custody of the couple to undefined

240

and unreasonable limits. Consequently, there is no reason to assume that the mere use or enjoyment of dowry by other members of the household, would have the effect of passing the possession and control thereof Jointly to the Hindu Undivided Family a such." "50. In the aforesaid context, pointed reference must be made to the opening word 'whoever' of S.405 of the Code to highlight that the criminal law does not take ken of any proximity of relationship for the offence of breach of trust. "Whoever" would include within its ambit the parents-in-law, the brothers-in-law, sisters- in-law (and other close relations of the husband) of a Hindu wife provided that the basic ingredients of entrustment or passing of dominion over her separate individual property stands fully satisfied. Apart from the peculiarity of the conjugal relationship and the consequent sharing of the matrimonial home, the existence of the blood relationship of the parties does not seem to be relevant for the applicability or otherwise of S.406 of the Code, Since the other members of the Hindu Joint family, to which the husband may belong, would not be covered by the presumption of jointness of custody v and possession of their individual properties by the spouses alone, they cannot by the mere fact of kinship be excluded from the scope of ss. 405 and 406 of the Code."

"56. To conclude, it necessarily follows from the aforesaid discussion that the very concept of the matrimonial home cannotes a jointness of possession and custody by the spouses even with regard to the movable properties exclusively owned by each of them. It is, therefore, inapt to view the same in view of the conjugal relationship as involving any entrustment or passing of dominion over property day-to-day by the husband to the wife or vice versa. Consequently, barring a special written agreement to the contrary, no question of any entrustment or dominion over property would normally arise during coverture or its imminent break-up. There fore, the very essential pre-requisites and the core ingredients of the offence under S.406 of the Penal Code would be lacking in a charge of criminal breach of trust of

241

property by one spouse against the other. Inevitably, therefore, the purported allegations of breach of trust betwixt husband and wife so long as the conjugal relation ship lasts and the matrimonial home subsists, cannot constitute an offence under Section 406 of the Indian Penal Code, subject to any special written agreement. Equally, as against the close relations of the husband, no facile presumption of entrustment and dominion over the dowry can be raised prims facie and this inevitably has to be by a subsequent conscious act of volition which must be specifically alleged and conclusively established by proof. Lastly, because of the definition in S. 2 of the Dowry Prohibition Act, the offences under the said Act cannot come within the ambit of S. 406 of the Indian Penal Code as these cannot stand together on the same set of facts." "57. Hence the answer (to the question) posed at the very outset is rendered in the affirmative. The bond of matrimony, therefore, bar the spectre of the criminal breach of trust qua the property of the spouses at the very threshold of the matrimonial home. It cannot enter its hallowed precincts except through the back door of a special written contract to the contrary with regard to such property."

I have extracted above several passages from the Judgment of the learned judges of the Full Bench in Vinod Kumar's case (supra) since I share their view and concern for peace and harmony in matrimonial homes and feel that the learned Single Judge who has quashed the wife's complaint in the present case was justified in relying heavily upon that judgment of the Full Bench. In these circumstances, 1 think that in the absence of a separate agreement and specific entrustment by the wife to the husband and of his relations and vice versa of the property of the husband to the wife and or her relation, it would not be possible to draw an inference of entrustment of custody or dominion over the property of one spouse to the other and his or her relations so as to attract the stringent provisions of s.406 I.P.C. The offense of criminal breach of trust is cognizable and non-bailable and punishable with imprisonment for a term of three years or

242

with fine or with both. In the absence of such a separate agreement for specific entrustment of the property of either spouse the appropriate remedy would appear to be by way of a civil suit where there is scope for the parties to the marriage coming together at the instance of relations, elders and well-wishers and patching up their differences. Entertaining complaints of the irate wife or husband against the husband or wife without even an allegation of a specific and separate agreement constituting entrustment of the property of the wife or the husband would have disastrous effects and consequences on the peace and harmony which ought to prevail in matrimonial homes. It is seen from para 45 of-the judgment in Vinod Kumar's case (supra) that in spite of diligent research no instance of any case of successful prosecution of the husband of wife at the instance of the wife or the husband could be brought to the notice of the learned Judges. It may be stated that none was brought to the notice of this Court either in the course of the arguments in this appeal. This would show that the spouses had not lightly rushed in the past to criminal courts with complaints of criminal breach of trust against the other spouses though in the day-to-day life there must have been numerous instances where the wife had used the property or cash of the husband for purposes different from the one for which they were given by the husband to be applied by the wife and vice-versa. I am anxious that no light-hearted change should be brought about in-the position and that the minimum requirement in such cases is a specific separate agreement whereby the property of the wife to husband was entrusted to the husband or wife and or his or her close relations. In the absence of such a specific separate agreement in the complaint, in the present case, I am of the opinion that the learned Single Judge was perfectly justified in following the decision of the Full Bench in Vinod Kumar s case (supra) and quashing the wife's complaint filed against the husband and his close relations. I would, therefore, dismiss the appeal.

In view of the majority decision, this appeal is allowed, the judgment of the High Court is set aside and the complaint filed by the appellant is restored. The accused may now be summoned and put on trial in accordance with law. S.R. Appeal allowed

NGOKC (pm)     04 August 2013

household items bought by husband post or pre marriage is not stridhan

Victim of Greed (Worker)     13 August 2013

Hello Nadeem ji,

I have gone thru the link you sent me about definition of Stridhan. Will you pleas elaborate on this para :-


Besides the ornaments and trousseau given at marriage, stridhan also includes all the gifts of money, property, jewellery and so on received by the woman before, during and after marriage from her family, her husband’s family, friends and even strangers.


 

Other experts says that anything bought as gift in form of jewellery or any household after marriage does not come under stridhan.

Victim of Greed (Worker)     13 August 2013

Hello Experts,

Tomorrow, I have cross examination for return of Stridhan at the family court. In my forum, Helping Hand had replied :-

Gifts etc [jwellery, ac, bus, car, window ac, split ac, almirah, washing machine, dishwasher etc etc] given at the time of marriage = stridhan. After marriage anything husband has brought, then it is not stridhan. You need not give back such items, if your wife has bills also she cannot claim it as stridhan. simple.

Whereas, in the link sent by Mr. Nadeem states the following :-

Besides the ornaments and trousseau given at marriage, stridhan also includes all the gifts of money, property, jewellery and so on received by the woman before, during and after marriage from her family, her husband’s family, friends and even strangers.

So, what is the exact definition of stridhan ?

As per Helping hand's opinion "You need not give back such items, if your wife has bills also she cannot claim it as stridhan. simple.".

But what if one jewellery is bought for her, in her name after 1 year of marriage and she has the bill ? Does it come under Stridhan ?

Is there any judgement where definition of Stridhan is mentioned ?


(Guest)


 

agar aapne ghar ke liye kuch khareeda ho, shaadi ke baad woh streedhan nahi hosakta aur aapki patni unn cheezon ko claim nahi karsakti.

 

but, she can claim what all given by her mom, her friends, your parents, and also strangers when? 1. during time or marriage, 2. before marriage. 3 after marriage.

 

 

But she cannot claim what you brought for your house for your own day to day use.. got it?


If you brought jwellery/car/property/flat etc etc for wife, then it is hers, she can claim it.

Adv k . mahesh (advocate)     13 August 2013

about different point in section 14 of hindu marriage act regarding stridhan and her right on the property

https://www.498a.org/contents/general/streedhan.pdf

this page clears you the doubt wheater husband side gifts are included in stridhan or not 

\https://www.mehnat.in/406_stridhan.html

this page gives the light of women right on her stridhan if husband is denying her right to claim the stridhan by judgement and steps 

https://www.indg.in/social-sector/socialwelfare-faqs/3.stridhan.pdf

there are number of judgement which clarify you about the what is stridhan and how to claim 

https://indiankanoon.org/search/?formInput=stridhan%20property%20

there are ample queries raised in our club about stridhan 

The word ''Stridhan ' is, derived from the words "stri " meaning woman and "dhana "meaning property. Essentially a word and concept, which comes down centuries from the Hindu smritis but has today, permeated all forms of marriages in all castes and religions.
The existence of Stridhan is an ingrained part of Indian culture from times of yore. As male dominated as the society may be, the existence of the custom of 'bride price' indicates that women understood the importance of financial independence and safeguarding their interests long before the feminist movement made it popular to do so.

Mitakshara II ix, 2 defines that stridhan means woman’s property. In the entire history of Hindu Law, woman’s rights to hold and dispose of property has been recognized.

The Dayabhaga School doesn’t recognize gifts of immovable property by husband as stridhan (Banerjee, Hindu Law of Marrriage and Stridhan, Pg 321).

Under all the schools of Hindu Law payments made to a Hindu female in lump sum or periodically for her maintenance and all the arrears of such maintenance constitute stridhan. Similarly, all movable or immovable properties transferred to her by way of an absolute gift in lieu of maintenance constitute her stridhan.

 

A full bench in Vinod Kumar Sethi v. State of Punjab AIR 1982 P& H 372 held that dowry and traditional presents made to a wife at the time of the marriage constitute her stridhan. 

 

Section 27 of the Hindu Marriage Act, 1955 says that in any proceeding under this Act, the Court may make such provisions in the decree as it thinks just and proper with respect to any property presented, at or about the time of marriage, which may belong jointly to both the husband and the wife. Refusal by husband to return the gift items given to wife at the time of marriage makes the husband liable for prosecution. The section does not bar the right of the aggrieved person to file criminal complaint under Section 406 I.P.C., if property belonging to the complainant is criminally misappropriated by the accused. The section empowers a Court while deciding a matrimonial dispute to also pass a decree in respect of property, which may jointly belong to both the husband and wife. This section at best provides a civil remedy to an aggrieved wife and does not in any way take away her right to file a criminal complaint if the property belonging to her is criminally misappropriated by her husband.

refer https://www.lawyersclubindia.com/forum/What-is-Stridhan--6777.asp#.UgnlN9KTTz4


Leave a reply

Your are not logged in . Please login to post replies

Click here to Login / Register